You are on page 1of 59

https://www.pdfnotes.

co/

VISION IAS
www.visionias.in

TEST-28
Test Booklet Series

GENERAL STUDIES (P) 2023 – Test – 3812


C
Time Allowed: Two Hours Maximum Marks: 200

LENGTH
FULL(COMPLETE TEST
SYLLABUS)
S

FULL LENGTH TEST (COMPLETE SYLLABUS):


Indian Constitution, Political System and
Governance + Geography + History of India + I
Indian Economy & Social Development
Ecology & Environment General Science
+ Current Affairs

2023
https://www.pdfnotes.co/

.
1. In the context of the Indian parliament, which 4. ‘Fabled for centuries for its blue waters and
of the following will lapse on the dissolution sparkling fish, laps the shores of Istanbul. It
of Lok Sabha? connects the Black Sea to the Aegean Sea. It
1. A bill passed by the Lok Sabha but
has witnessed the largest outbreak of ‘sea
pending in the Rajya Sabha
snot, also known as ‘marine mucilage’, it is a
2. A bill pending in the Rajya Sabha but not
thick, slimy grey-brown sheet that is formed
passed by the Lok Sabha
3. All pending assurances that are to be by dead and living organic material. The

examined by the Committee on growth of the mucilage, which floats up on


Government Assurances the surface of the sea like a brown phlegm, is
Select the correct answer from the code given posing a severe threat to the marine
below. ecosystem’.
(a) 1 only
Identify the waterbody mentioned in the
(b) 1 and 2 only
above description.
(c) 1 and 3 only
(a) Mediterranean Sea
(d) 2 and 3 only
(b) Bering Sea

2. With reference to Buddhism, which of the (c) The Sea of Marmara


following statements is not correct? (d) Caspian Sea
(a) There are three principal elements in
Buddhism: Buddha, dhamma, and 5. Consider the following statement with
sangha.
reference to the Bay of Bengal Initiative for
(b) Buddhism rejects the authenticity of the
Multi-Sectoral Technical and Economic
Vedas.
Cooperation (BIMSTEC):
(c) Tipitakas have been written in prakrit
1. This organization came into force
language.
(d) In various Buddhist texts, Buddha is also through Bangkok declaration in 1997.

known as Tathagata and Sakyamuni. 2. The members of BIMSTEC include


Bangladesh, India, Myanmar, and
3. Which of the following statements best Thailand only.
describes Pigovian Tax? 3. The member countries of BIMSTEC
(a) It is a tax placed on any good which
constitute more than 50% of the world’s
creates negative externalities.
GDP.
(b) It is a tax levied on purchase and sale of
Which of the statements given above is/are
securities.
(c) It is a tax levied upon digital assets correct?

legally recognised in India. (a) 1 only


(d) It is a tax that unlisted startups are (b) 2 only
required to pay on their additional (c) 1 and 2 only
earnings. (d) 1, 2 and 3
2 www.visionias.in ©Vision IAS
https://www.pdfnotes.co/

.
6. Which of the following may be considered 8. Consider the following statements regarding
digital public goods? Open Radio Access Network (Open-RAN)
Technology, frequently seen in the news:
1. Aadhar
1. Open RAN Technology allows
2. Digilocker transmission and receiving of signals
3. Central KYC from smartphones and other devices.
4. Account aggregator 2. The Open RAN architecture allows for
the separation between hardware and
Select the correct answer using the code given
software with open interfaces.
below. 3. Open RAN would reduce a telecom
(a) 1 and 2 only operator’s network deployment cost as it
is interoperable with other networks such
(b) 3 and 4 only
as 4G.
(c) 1 only
Which of the statements given above are
(d) 1, 2, 3 and 4 correct?
(a) 1 and 2 only
(b) 2 and 3 only
7. With reference to the Provisions of the
(c) 1 and 3 only
Panchayats (Extension to the Scheduled (d) 1, 2 and 3
Areas) Act, 1996 (PESA), consider the
following statements: 9. Consider the following statements regarding
"Apis karinjodian (Indian Black honey Bee)":
1. Only persons belonging to a tribal
1. It is a new endemic honey bee species
community can contest for posts of recently discovered in the Western Ghats.
Chairpersons of Panchayati Raj 2. It has the ability to produce higher
Institutions in the areas covered under quantities of honey which is thicker in
consistency.
PESA.
3. It is listed as endangered under the IUCN
2. The main objective of the PESA Act is to Red list.
extend the provisions of Part IX of the Which of the statements given above are
Constitution of India to areas of the Sixth correct?
(a) 1 and 2 only
Schedule of the Constitution.
(b) 2 and 3 only
3. Under PESA Act the recommendation of (c) 1 and 3 only
Gram Sabha is mandatory for granting (d) 1, 2 and 3

mining lease for minor minerals in the


10. Recently, the Union Cabinet approved the
Scheduled Areas.
ratification of three protocols of Chicago
Which of the statements given above is/are Convention. In this context, Chicago
correct? Convention is related to which of the
following sectors?
(a) 1 and 2 only
(a) Digital Infrastructure
(b) 2 and 3 only (b) Public Health
(c) 3 only (c) Civil Aviation
(d) 1 and 3 only (d) Agricultural export
3 www.visionias.in ©Vision IAS
https://www.pdfnotes.co/

.
11. With reference to the Individual Privileges of 14. Consider the following statements regarding
the members of the Parliament, consider the "Refuse-Derived Fuel (RDF)":
following statements:
1. It is a fuel produced from carbon capture,
1. A member of the Parliament is not
utilization and storage (CCUS)
protected from civil proceedings for
his/her speech in the Parliament after the technology.
expiration of 40 days of a session of the 2. It is considered a renewable energy
Parliament. source.
2. A member of Parliament can be arrested
3. It can be used as an alternative fuel to
during the session of Parliament in
preventive detention cases. fossil fuels in automobiles.
Which of the statements given above is/are Which of the statements given above is/are
correct? correct?
(a) 1 only (a) 2 only
(b) 2 only
(b) 1 and 2 only
(c) Both 1 and 2
(d) Neither 1 nor 2 (c) 1 and 3 only
(d) 1, 2 and 3
12. Which of the following are the applications of
'technical textiles'?
15. Consider the following statements regarding
1. Bullet Proof Jackets
Indian agriculture trade during the last
2. Fishing nets
3. Artificial turfs in sports decade:
4. Medical implants 1. India's import of agricultural products has
5. Airline disposables declined over the period.
Select the correct answer using the code given
2. India's trade surplus in agriculture trade
below.
has been increasing steadily.
(a) 2, 3, 4 and 5 only
(b) 3 and 4 only Which of the statements given above
(c) 1, 2 and 5 only is/are correct?
(d) 1, 2, 3, 4 and 5 (a) 1 only
(b) 2 only
13. You are walking along the McMohan Line
from west to east (c) Both 1 and 2
1. If you jump back to the Indian side, you (d) Neither 1 nor 2
will find yourself in Sikkim.
2. The latitude passing from your location
16. The term 'Nucleoside Reverse Transcriptase
will cut through Nagaland and Rajasthan.
Inhibitor (NRTI)' is often talked about in the
3. At the end of your journey along the line,
you may end up in Myanmar. context of
Which of the statements given above is/are (a) genes introduced in the genetically
correct? modified plants
(a) 1 only
(b) development of antimicrobial resistance
(b) 1 and 2 only
(c) anti retroviral therapy
(c) 2 and 3 only
(d) 3 only (d) use of food preservatives
4 www.visionias.in ©Vision IAS
https://www.pdfnotes.co/

.
17. With reference to Kailashnath Temple 19. Consider the following statements with

situated at Ellora, consider the following reference to the Employees' Provident Fund
Organisation (EPFO):
statements:
1. It manages social security funds of
1. It is a rock-cut shrine within a rectangular
workers in the organized and semi-
court built in nagara style.
organized sectors in India.
2. It features sculptural representations of 2. It has a Universal Account Number for
both Shiva and Vishnu. the portability of Provident Fund

3. It is a UNESCO World Heritage Site. accounts of its members.


3. It is under the administrative control of
Which of the statements given above is/are
the Ministry of Finance.
correct?
Which of the statements given above is/are
(a) 1 only
correct?
(b) 1 and 3 only (a) 1 only
(c) 2 and 3 only (b) 1 and 2 only

(d) 1, 2 and 3 (c) 2 and 3 only


(d) 1, 2 and 3

18. Which of the following can be the likely


20. With reference to the Anglo-Mysore rivalry,
reason for land subsidence i.e. sinking of the
consider the following statements:
ground because of underground material 1. The English abolished the state of
movement? Mysore through the treaty of

1. unplanned construction Srirangapattinam.


2. The Treaty of Madras was signed
2. over-population
between Haider Ali and the British after
3. obstruction of the natural flow of water
the first Anglo-Mysore war.
4. hydel power activities
3. Lord Cornwallis commanded the British
5. reactivation of a geographic fault forces during the first Anglo- Mysore
Select the correct answer using the code given war.

below. Which of the statements given above is/are


correct?
(a) 1, 3, 4 and 5 only
(a) 1 only
(b) 2, 3 and 4 only
(b) 2 and 3 only
(c) 1, 2 and 5 only
(c) 2 only
(d) 1, 2, 3, 4 and 5 (d) 1 and 3 only
5 www.visionias.in ©Vision IAS
https://www.pdfnotes.co/

.
21. Which of the following demands was not 24. Arrange the following events of the revolt of
made by the Muslim League under its 'Delhi 1857 in a chronological sequence:
1. Azamgarh Proclamation by the Rebels
Proposals'?
2. Deceleration of Bahadur Shah Zafar as
(a) Formation of three new Muslim majority
Shahenshan-e-Hindustan
provinces— Sindh, Baluchistan and
3. Recapture of Delhi by the Britishers
North-West Frontier Province 4. Mutiny of Sepoys at Meerut
(b) One-third representation to Muslims in Select the correct answer using the code given
Provincial Legislative Assembly below.
(a) 2-4-3-1
(c) Representation to Muslims in Punjab and
(b) 1-4-2-3
Bengal in proportion to their population
(c) 4-1-2-3
(d) Joint electorates in place of separate
(d) 4-2-1-3
electorates with reserved seats for
Muslims 25. In the context of the Indian parliament,
consider the following statements regarding

22. Consider the following statements regarding the office of the whip:
1. The Ministry of Parliamentary Affairs
Net Interest Margins (NIM):
organises the All India Whips
1. It is the difference between the interest
Conference
income earned and the interest paid by a 2. The Government chief whip acts as a
bank relative to its interest-earning assets. communication link between the leader
2. NIM increases when the demand for of the House and the members of the
ruling party.
savings increases relative to the demand
3. The chief whip of a political party
for loans.
regulates and monitors the behavior of
Which of the statements given above is/are
members of the party in the Parliament.
correct? Which of the statements given above are
(a) 1 only correct?
(b) 2 only (a) 1 and 2 only

(c) Both 1 and 2 (b) 2 and 3 only


(c) 1 and 3 only
(d) Neither 1 nor 2
(d) 1, 2 and 3

23. Which one of the following is the context in 26. 'Operation Barkhane' recently seen in the
which the terms 'minisatellites' and news is a/an
'Restriction fragment length polymorphism' is (a) anti-piracy operation in Indian ocean

mentioned? region.
(b) counter-insurgency operation in all the
(a) Analysis of impact of microgravity on
Sahel region countries of Africa.
astronauts
(c) evacuation program for the recent
(b) Production of biofertilizers earthquake in Turkiye.
(c) DNA fingerprinting (d) an operation to send counter-terrorist
(d) Spread of viral diseases force to Afghanistan.
6 www.visionias.in ©Vision IAS
https://www.pdfnotes.co/

.
27. Consider the following statements: 30. Consider the following statements regarding
1. Frictional unemployment is generally the "Global Carbon Budget Report":
caused by fundamental shifts in an 1. It is released by Global Environment
Facility.
economy.
2. The latest data from its 2022 report
2. Recent layoffs in tech companies will
confirm that the rate of increase in fossil
lead to an increase in seasonal CO2 emissions has slowed compared to
unemployment. past decade.
Which of the statements given above is/are Which of the statements given above is/are
correct? correct?
(a) 1 only
(a) 1 only
(b) 2 only
(b) 2 only
(c) Both 1 and 2
(c) Both 1 and 2 (d) Neither 1 nor 2
(d) Neither 1 nor 2
31. Due to their unique chemical properties,
28. 'Bor Thekera' recently seen in the news is a/an semiconductors play a prominent role in the
(a) early human species development of the electronics industry.
What happens when a semiconductor is
(b) medicinal plant found to have
doped with impurities like boron and
cardioprotective potential
aluminum?
(c) fossils of a kind of dinosaur (a) Its electrical conductivity increases.
(d) traditional water harvesting method (b) It shows a positive temperature
coefficient of resistance.
29. Consider the following statements regarding (c) It becomes dipolar in nature.
(d) Its electrical resistivity increases.
the Swarajists and No-Changers:
1. C. Rajagopalachari resigned from the
32. Recently, revised guidelines to broaden the
presidentship of the Congress and formed scope of the Member of Parliament Local
the Congress-Khilafat Swarajya Party. Area Development Scheme (MPLADS) have
2. Responsivists were a group within the no- been released. In this context consider the
changers who wanted to work the following statements regarding the
legislative reforms and hold office MPLADS:
1. A new web portal was launched for the
wherever possible.
implementation of the revised fund flow
3. In the Belgaum session, Congress
procedure under MPLADS.
allowed Swarajist to carry on work in the 2. Under the scheme, both elected and
legislature on their behalf. nominated members of Parliament can
Which of the statements given above is/are recommend developmental work.
correct? Which of the statements given above is/are
correct?
(a) 2 and 3 only
(a) 1 only
(b) 3 only
(b) 2 only
(c) 1 and 2 only (c) Both 1 and 2
(d) 2 only (d) Neither 1 nor 2
7 www.visionias.in ©Vision IAS
https://www.pdfnotes.co/

.
33. With reference to the appeal by Special 35. Consider the following statements regarding
the “Loss and Damage Fund”
Leave, consider the following statements:
1. It aims to provide financial assistance to
1. It is a discretionary power of the supreme nations most vulnerable and impacted by
court granted under Article 136 of the the effects of climate change.
2. The Fund has envisaged a goal for
Indian constitution.
Developed countries to mobilize USD
2. It can only be granted against the final 100 million per year by 2025 to support
judgments of the courts and Tribunals climate action in developing countries.
Which of the statements given above is/are
and not interlocutory decisions.
correct?
3. The appeals under this special provision (a) 1 only
must be related to constitutional matters (b) 2 only
(c) Both 1 and 2
and not to matters like income tax and (d) Neither 1 nor 2
revenue.
36. The Mayor of the Municipal Corporation of
Which of the statements given above is/are
Delhi is:
correct? (a) Directly elected by the people of Delhi
(a) 1 only (b) Nominated by the Lt Governor from the
party securing majority vote in the
(b) 1 and 2 only
corporation
(c) 2 and 3 only (c) Elected by the Councillors of Municipal
(d) 1 and 3 only Corporation of Delhi
(d) Nominated by the President of India

34. With reference to “Sealed cover 37. Recently physicists tested an electron’s
response to a magnetic field with extreme
jurisprudence”, a term sometimes seen in the
accuracy which helps in the analysis of
news, consider the following statements: Standard Model of Particle Physics. In this
1. It prevents parties involved in the case context, consider the following statements
with respect to Standard Model:
from having a full overview of the
1. It is a standard theory that explains how
charges against them. sub atomic particles like quarks and
2. Only the Chief Justice of India has the leptons make up the matter.
2. Einstein explained the Standard Model in
power to direct any document to be kept
his General Theory of Relativity.
confidential in a sealed cover. 3. The model accounts for the dark matter
and could explain why universe has more
Which of the statements given above is/are
matter than anti-matter.
correct? Which of the statements given above is/are
(a) 1 only correct?
(a) 1 only
(b) 2 only
(b) 2 and 3 only
(c) Both 1 and 2 (c) 1 and 2 only
(d) Neither 1 nor 2 (d) 1 and 3 only
8 www.visionias.in ©Vision IAS
https://www.pdfnotes.co/

.
38. Which one of the following rulers honoured 41. Consider the following statements with
Jain scholar Jinaprabha Suri in his court? reference to hypermetropia:
(a) Ghiyas ud din Balban 1. A person with hypermetropia can see
(b) Mohammad bin Tuglaq
nearby objects clearly but cannot see the
(c) Alauddin Khalji
distant objects clearly.
(d) Mubarak Shah
2. Hypermetropia arises due to the

39. Consider the following statements regarding elongation of the eyeball.

industrial corridors in India: 3. Convex lens is used to correct the defect.


1. Under the National Industrial Corridor Which of the statements given above is/are
Programme, the government has correct?
approved the development of 111 (a) 1 only
industrial corridors.
(b) 1 and 2 only
2. Tumakuru Industrial Township project is
(c) 3 only
being built under Vizag-Chennai
(d) 2 and 3 only
industrial corridor.
3. NITI Aayog oversees the unified
development of all the industrial 42. Consider the following statements regarding
corridors. Indian Forest Act, 1927:
Which of the statements given above is/are 1. The government regulates timber cutting
correct?
and cultivation in private forest lands.
(a) 1 and 2 only
2. The State Government can give the rights
(b) 2 and 3 only
to any village group, which the
(c) 1 only
(d) None Government has over any land which has
been reserved.
40. Consider the following differences between Which of the statements given above is/are
Chief Secretary and Cabinet Secretary: correct?
1. The chief secretary is the administrative (a) 1 only
head of the state secretariat, while the
(b) 2 only
cabinet secretary is the administrative
(c) Both 1 and 2
head of the central secretariat.
(d) Neither 1 nor 2
2. While the chief secretary is not chief of
state secretaries, the cabinet secretary is
the chief of central secretaries. 43. With reference to India, the terms ‘rekhapida,
Which of the statements given above is/are pidhadeul and khakra’ pertain to
correct? (a) song and dance performance
(a) 1 only (b) martial arts in South India
(b) 2 only
(c) rock-cut cave paintings
(c) Both 1 and 2
(d) architectural features of temples
(d) Neither 1 nor 2
9 www.visionias.in ©Vision IAS
https://www.pdfnotes.co/

.
44. Which of the following statements best 47. Which of the following statements is not
describes 'RCH-ANMOL'? correct?
(a) It is an application to track pregnant (a) Dawn and twilight stay for longer time in
women, infants and children for their the temperate regions than the tropics.
nutrition status. (b) Generally, on 21st September, the Sun is
(b) It is a type of fluid replacement to treat
vertically overhead at the tropic of
dehydration.
capricorn.
(c) It is a portal for mineral block auction
(c) While the latitudes are equally spaced,
under the Ministry of Mines.
distance between longitudes vary.
(d) It is a campaign to create awareness
(d) Seasonal extremes in two hemispheres
regarding pneumonia in children.
are impacted by earth's axial precession.
45. With reference to the Vaikom Satyagraha,
1924, consider the following statements: 48. With reference to the recently launched
1. It aimed to secure the rights of 'Sagar Parikrama', consider the following
untouchables for using the forbidden statements:
roads to the temple. 1. It was launched by the Ministry of Ports,
2. Periyar E.V. Ramaswamy provided Shipping, and Waterways.
leadership to the Vaikom Satyagraha. 2. One of its main objectives includes the
3. It was launched during the course of the facilitation of interaction with fishermen
Non-cooperation Movement launched by and promoting responsible fisheries.
Mahatma Gandhi.
Which of the statements given above is/are
Which of the statements given above is/are
correct?
correct?
(a) 1 only
(a) 1 only
(b) 2 only
(b) 1 and 2 only
(c) Both 1 and 2
(c) 2 only
(d) 1, 2 and 3 (d) Neither 1 nor 2

46. Recently, the Reserve Bank of India (RBI) 49. Consider the following statements regarding
has increased the policy rate. In this context, the "Great Blue Wall Initiative":
which of the following is/are the likely 1. It is an international movement that hopes
impacts of the increase in repo rate by the to create interconnected protected marine
RBI? areas to counteract the effects of climate
1. Increase in income from fixed deposits change and global warming.
2. Increase in demand in the real estate 2. The initiative was formally launched
sector
during the COP 26 summit in Glasgow.
3. Decrease in cost of borrowing
Which of the statements given above is/are
Select the correct answer using the code given
correct?
below.
(a) 1 only
(a) 1 and 2 only
(b) 2 only
(b) 2 and 3 only
(c) 1 only (c) Both 1 and 2

(d) 1, 2 and 3 (d) Neither 1 nor 2

10 www.visionias.in ©Vision IAS


https://www.pdfnotes.co/

.
50. Consider the following pairs: 52. Which of the following statements is/are
correct regarding the Indus valley
Commission Objective
civilization?
1. Islington : Examine the Indian 1. People performed uniform religious and
Commission, demand for reforms in the funerary practices.
1912 Public Service of British 2. The practice of navigation along the coast
of the Arabian Sea was prevalent.
India
3. They produced rice and domesticated
2. Muddiman : Inquired into the famines dogs.
Commission, and crisis of Indian Select the correct answer using the code given
below.
1924 agriculture
(a) 1 only
3. Lee : To inquire into the (b) 1 and 2 only
Commission, conditions and prospects (c) 2 and 3 only
(d) 1, 2 and 3
1923 of universities in India

How many pairs given above are correctly 53. Consider the following pairs:
matched? Places in news Location
(a) None of the pairs 1. Kahramanmaras : Turkiye
2. Damascus : Syria
(b) Only one pair
3. Tokara island : Japan
(c) Only two pairs
Which of the pairs given above are correctly
(d) All three pairs matched?
(a) 1 and 2 only
(b) 1 and 3 only
51. Which of the following constitutional and
(c) 2 and 3 only
legal provisions in India are related to NGOs?
(d) 1, 2 and 3
1. Seventh Schedule of the Indian
constitution 54. Consider the following statements regarding
the Rotterdam Convention:
2. Article 19 of the Indian constitution
1. Annex III of the includes chemicals that
3. The Foreign Contribution Regulation have been banned or severely restricted
Act, 2010 for health or environmental reasons by

4. The Societies Registration Act, 1860 two or more parties.


2. Under the convention two chemicals,
5. The Companies Act, 2013
Iprodione and Terbufos are banned to be
Select the correct answer from the code given used under all circumstances.
below. Which of the statements given above is/are
correct?
(a) 1 and 5 only
(a) 1 only
(b) 2, 3, 4 and 5 only (b) 2 only
(c) 1, 2, 3 and 4 only (c) Both 1 and 2
(d) 1, 2, 3, 4 and 5 (d) Neither 1 nor 2
11 www.visionias.in ©Vision IAS
https://www.pdfnotes.co/

.
55. Consider the following statements with 58. Consider the following statements with
respect to phonons: reference to Asian Development Bank:
1. Phonons are building blocks of neutrons. 1. It is a Beijing-based development bank
2. Phonons do not have electric charge and that deals with development challenges
cannot be deflected by the magnetic field. facing our world, such as climate change
Which of the statements given above is/are and water security.
correct? 2. China and India are its largest
(a) 1 only shareholders with twenty percent share
(b) 2 only each.
(c) Both 1 and 2 3. Its membership is not restricted to Asia.
(d) Neither 1 nor 2 Which of the statements given above is/are
correct?
56. Consider the following pairs: (a) 1 and 2 only
Books Authors (b) 3 only
1. Karpuramanjari : Rajashekhara (c) 1, 2 and 3
2. Kathasaritsagara : Somadeva (d) 2 only
3. Brihatkatha : Gunadhya
How many pairs given above are correctly 59. Consider the following statements regarding
matched? White Label ATMs (WLAs):
(a) None of the pairs 1. White-label ATMs are owned and
(b) Only one pair operated by a non-bank.
(c) Only two pairs 2. There is no interchange fee on
(d) All three pairs transactions done using WLAs.
3. The Reserve Bank of India (RBI)
57. Which of the following statements is/are authorizes white-label ATM operators
correct? under the Banking Regulations Act 1949.
1. The state with the largest area in India Which of the statements given above is/are
also has the biggest national park. correct?
2. The state with the longest mainland (a) 1 and 2 only
coastline in India also has the only (b) 2 and 3 only
habitation of Asiatic lions in the wild. (c) 1 only
3. The state with the lowest population
(d) 1 and 3 only
density in India also hosts the highest
number of Particularly vulnerable tribes
60. Which of the following puranas is considered
Select the correct answer using the code given
a sourcebook of Indian painting?
below.
(a) Markandeya Purana
(a) 2 only
(b) Vishnudharmottara Purana
(b) 3 only
(c) Agni Purana
(c) 1 and 3 only
(d) Padma Purana
(d) 1 and 2 only
12 www.visionias.in ©Vision IAS
https://www.pdfnotes.co/

.
61. With reference to the early phase of the Indian 63. Consider the following statements with
National Congress (INC), consider the respect to Alternate Dispute Resolution:
1. Arbitration is an attempted resolution of
following statements:
issues raised by a complaint, or by the
1. Since the inception of INC, government
investigation of a complaint, through
officials were not allowed to attend its informal negotiations.
sessions. 2. Mediation is where the opposing parties
2. The moderate leadership in the Indian are brought together to work out a
settlement or the agreement that both
National Congress did not believe in
parties accept or reject.
social reforms.
3. Conciliation is more formal than
3. The early leadership of INC demanded a mediation and has a lot of similarities
reduction of land revenue and protection with traditional court proceedings.

of peasants from unjust landlords. Which of the statements given above is/are
not correct?
Which of the statements given above is/are
(a) 1 and 2 only
correct? (b) 2 only
(a) 1 and 3 only (c) 1 and 3 only
(b) 1 and 2 only (d) 1, 2 and 3

(c) 3 only
64. Which of the following best describes
(d) 1, 2 and 3
petrodollars recently seen in the news?
(a) US dollars paid to oil exporting countries
62. Consider the following statements with (b) The currencies of those West Asian and
respect to Quantum dots: African countries that export oil
(c) Cryptocurrencies used by South
1. They are nanocrystals of semiconducting
American countries
material having unique optical and
(d) US dollar-denominated deposits at
electronic properties. foreign banks or at the overseas branches
2. They are capable of emitting of American banks
monochromatic light when excited by an
65. The Mauna Loa volcano eruption, recently
energy source.
seen in the news. Consider the following
3. They cannot be manufactured at room
statements regarding Mauna Loa Volcano:
temperature as their manufacturing 1. It is the world's largest active Volcano.
process involves plasma synthesis. 2. It is a type of Composite Volcano.
Which of the statements given above is/are 3. It is located on the island of Hawaii.
Which of the statements given above is/are
correct?
correct?
(a) 1 and 2 only
(a) 2 only
(b) 1 only (b) 2 and 3 only
(c) 2 and 3 only (c) 1 and 3 only

(d) 1 and 3 only (d) 1, 2 and 3

13 www.visionias.in ©Vision IAS


https://www.pdfnotes.co/

.
66. Consider the following pairs: 69. Giving to Amplify Earth Action (GAEA), a
Turtle IUCN Status global initiative to finance net zero actions is
1. Ganges Softshell : Critically
launched by which of the following
Turtle Endangered
2. Red Crowned : Endangered institutes?
roofed Turtle (a) United Nations Environment Program
3. Indian Black Turtle : Least concern
(b) World Bank
Which of the pairs given above is/are
correctly matched? (c) World Economic Forum
(a) 1 and 2 only (d) Global Environment Facility
(b) 2 and 3 only
(c) 3 only
(d) 1, 2 and 3 70. Recently, Memorandum of Understanding
(MoU) was signed between Information
67. Consider the following statements regarding
Fusion Centre for Indian Ocean Region (IFC-
Kelp forests recently seen in the news:
1. They are underwater plant species IOR) and the Regional Coordination
ecosystems formed in shallow waters. Operations Centre (RCOC), Seychelles. In
2. Kelp forests grow mostly in equatorial this context, consider the following
regions.
statements about Information Fusion Centre
3. Giant Kelp harvested from kelp forests is
used as a binding material in the food for Indian Ocean Region (IFC-IOR):
industry. 1. It was established by the Indian Navy to
Which of the statements given above is/are
keep track of shipping traffic.
correct?
(a) 1 only 2. Its headquarters is situated in India.
(b) 1 and 2 only 3. It provides information related to weather
(c) 2 and 3 only
forecasts and warnings.
(d) 3 only
Which of the statements given above is/are
68. Consider the following statements with correct?
reference to the non-aligned movement:
(a) 1 only
1. The concept of NAM originated at the
Asia-Africa Bandung conference held in (b) 1 and 2 only
Indonesia. (c) 2 and 3 only
2. NAM has a permanent secretariat in (d) 1, 2 and 3
Cairo, Egypt.
3. India, Ghana, Egypt, and Yugoslavia
were their founding member countries. 71. Which one of the following peasant struggles
Which of the statements given above is/are was an outcome of the Ijaradari system?
correct?
(a) Birsaite Ulgulan (1899-1900)
(a) 1 only
(b) 1 and 3 only (b) Maratha Peasant Uprising (1875)
(c) 2 and 3 only (c) Phulaguri Dhawa (1861)
(d) 1, 2 and 3
(d) Rangpur uprising (1783)
14 www.visionias.in ©Vision IAS
https://www.pdfnotes.co/

.
72. Generally, there is a lack of cyclones during 75. Which of the following statements is not
the main monsoon months of July and August correct with respect to human immune
in India. What could be the possible reason? system?
(a) Low ocean surface temperatures in Bay (a) Neutrophils are capable of ingesting

of Bengal particles of microorganisms thus

(b) Lack of pre-existing near surface preventing infection.

disturbance around Indian Ocean (b) Human leukocyte antigen (HLA) system

(c) High vertical wind shear in the region helps the body in differentiating the body

(d) Lack of formation of fronts cells from foreign cells.


(c) B cells respond to pathogens by

73. Which of the following are excluded from the producing large quantities of antibodies
to neutralize foreign objects like bacteria.
Divisible Pool of central taxes?
(d) T cells develop into plasma cells that
1. Cost of collection of taxes
make antibodies to fight infection.
2. Cess and Surcharge
3. Taxes and duties collected by Centre
76. With reference to India, Ghoghla,
4. National Calamity Contingent Duty
Rushikonda, Kovalam, and Kadmat are the
Select the correct answer using the code given
names of:
below.
(a) turtle nesting sites
(a) 1, 2 and 3 only
(b) intensive fishing areas
(b) 1, 2 and 4 only
(c) beaches with ‘Blue Flag’ tag
(c) 1, 3 and 4 only
(d) ancient port cities
(d) 2, 3 and 4 only

77. Consider the following statements regarding


74. Arrange the following discontinuities inside
the transfer pricing regulations:
the Earth in sequential order from the inner
1. Transfer pricing (TP) is an accounting
core of the Earth to the surface.
practice that enables subsidiaries of the
1. Conrad Discontinuity
same company to transact with each
2. Lehmann Discontinuity
other.
3. Mohorovicic Discontinuity
2. Transfer pricing in India is governed
4. Gutenberg Discontinuity under the Companies Act, 2013.
Select the correct answer using the code given Which of the statements given above is/are
below. correct?
(a) 1-3-4-2 (a) 1 only
(b) 4-3-2-1 (b) 2 only
(c) 3-1-4-2 (c) Both 1 and 2
(d) 2-4-3-1 (d) Neither 1 nor 2
15 www.visionias.in ©Vision IAS
https://www.pdfnotes.co/

.
78. Unying Aran festival often seen in the news, 82. The first World Sloth Bear Day was observed
is a new year festival of which state? on October 12, 2022, to draw attention to the
(a) Uttarakhand
conservation and protection of sloth bears. In
(b) Manipur
this context, consider the following
(c) Arunachal Pradesh
(d) Meghalaya statements regarding sloth bears:
1. It is endemic to the Indian subcontinent.
79. Consider the following statements regarding 2. It is listed as ‘Endangered’ on the IUCN
components of light pollution:
Red List.
1. Skyglow which is the brightening of the
3. It is listed under Schedule I of The Wild
night sky over inhabited areas is
considered a component of light Life (Protection) Act of India, 1972.
pollution. Which of the statements given above are
2. Clutter which is bright, confusing, and correct?
excessive groupings of light sources are
(a) 1, 2 and 3
impacting the migration routes of birds.
(b) 1 and 2 only
Which of the statements given above is/are
correct? (c) 2 and 3 only
(a) 1 only (d) 1 and 3 only
(b) 2 only
(c) Both 1 and 2
83. In the context of Indian parliament, which of
(d) Neither 1 nor 2
the following statements best describes

80. What is common between 'Schoenorchis 'Kangaroo Closure'?


mishmensis' and 'Eulophia obtusa' ? (a) It is one when the undiscussed clauses of
(a) They are rare reptile species. a bill or a resolution are also put to vote
(b) They are orchid varieties.
along with the discussed ones due to want
(c) They are butterfly species.
of time.
(d) They are medicinal plants.
(b) The clauses of a bill or a lengthy
81. Consider the following statements with resolution are grouped into parts before
reference to mercury:
the commencement of the debate.
1. It is the only elemental metal that is liquid
(c) Only important clauses are taken up for
at room temperature.
2. It is used as a filling in dentistry. debate and voting and the intervening

Which of the statements given above is/are clauses are skipped over and taken as
correct? passed.
(a) 1 only
(d) It is one when a member moves that the
(b) 2 only
‘matter having been sufficiently
(c) Both 1 and 2
(d) Neither 1 nor 2 discussed be now put to vote’.

16 www.visionias.in ©Vision IAS


https://www.pdfnotes.co/

.
84. In the context of bad loans, arrange the 87. Consider the following statements regarding
following categories in the increasing order capital Intensive industry:
of risk. 1. Capital-intensive industries require a
1. Substandard Asset high percentage of investment in labor.
2. A capital-intensive production process
2. Doubtful Asset
will have higher labor productivity.
3. Loss Asset
Which of the statements given above is/are
4. Special Mention Account
correct?
Select the correct answer using the code given
(a) 1 only
below. (b) 2 only
(a) 4-1-2-3 (c) Both 1 and 2
(b) 4-2-3-1 (d) Neither 1 nor 2
(c) 3-2-1-4
(d) 3-4-2-1 88. Consider the following statements regarding
the Protection of Women from Sexual
85. With reference to the cultural history of India, Harassment (POSH) Act, 2013:
consider the following statements: 1. It was the result of the Vishakha
guidelines laid down by the Supreme
1. Pravrajya ceremony marked a person’s
Court in 1997.
going forth from home into
2. Every employer with 10 or more
homelessness.
employees is required to constitute an
2. An upasaka was a Buddhist follower who
Internal Complaints Committee.
had taken monastic vows. 3. The Internal Complaints Committee has
Which of the statements given above is/are the powers of a civil court.
correct? Which of the statements given above are
(a) 1 only correct?
(b) 2 only (a) 1 and 2 only
(c) Both 1 and 2 (b) 2 and 3 only
(d) Neither 1 nor 2 (c) 1 and 3 only
(d) 1, 2 and 3

86. Which of the following are included in the


89. Which of the following was/were included in
definition of 'State' as defined by Article 12
the Non-Cooperation movement launched in
of the Indian constitution?
1920?
1. Board of Control for Cricket in India
1. Surrender of titles and honorary offices
2. Deemed Universities 2. Boycott of British courts
3. Judiciary performing judicial role 3. Boycott of elections to provincial and
4. Local authorities Central assembly elections
Select the correct answer using the code given Select the correct answer using the code given
below. below.
(a) 1, 3 and 4 only (a) 1 only
(b) 1, 2, 3 and 4 (b) 2 and 3 only
(c) 2 and 4 only (c) 1, 2 and 3
(d) 2 only
(d) 3 and 4 only
17 www.visionias.in ©Vision IAS
https://www.pdfnotes.co/

.
90. With reference to Puisne Judges, consider the 92. Which of the following functionaries in the

following statements: Indian polity is/are elected through open


ballot voting?
1. The term ‘Puisne Judges’ is used to refer
1. Member of Rajya Sabha
to judges of the higher judiciary to denote 2. Member of the Legislative Council
seniority of rank. 3. President
4. Vice President
2. In India, the term is used for reference
Select the correct answer using the code given
during seniority-based appointments has below.
no impact on the judicial power of a (a) 1 and 2 only
(b) 1 only
judge.
(c) 3 and 4 only
Which of the statements given above is/are (d) 1, 2 and 3 only
not correct?
93. Consider the following statements regarding
(a) 1 only
“Healthy and Energy Efficient Buildings
(b) 2 only
Initiative”:
(c) Both 1 and 2 1. It is an initiative of Energy Efficiency

(d) Neither 1 nor 2 Services Ltd (EESL) jointly with the US


Agency for International Development
(USAID).
91. Consider the following statements with 2. The initiative will address the challenges
reference to the differences between of retrofitting existing buildings and air
conditioning systems.
Enforcement Directorate (ED) and the
Which of the statements given above is/are
Central Bureau of Investigation (CBI). correct?
1. Unlike CBI, ED cannot register a case on (a) 1 only
(b) 2 only
its own but can arrest a person under
(c) Both 1 and 2
Prevention of Money Laundering Act, (d) Neither 1 nor 2
2002.
94. Consider the following pairs:
2. While CBI functions under the Ministry
Tradition State
of Home Affairs, ED works under the
1. Theyyam : Kerala
Ministry of Finance. 2. Kambala : Andhra Pradesh
Which of the statements given above is/are 3. Yakshagana : Karnataka
correct? How many pairs given above are correctly
matched?
(a) 1 only
(a) Only one pair
(b) 2 only (b) Only two pairs
(c) Both 1 and 2 (c) All the three pairs
(d) None of the pairs
(d) Neither 1 nor 2
18 www.visionias.in ©Vision IAS
https://www.pdfnotes.co/

.
95. The Gross Domestic Climate Risk Report is 98. With reference to the medieval history of
released by which of the following India, Khuts and Muqaddams refer to
organizations? (a) religious leaders
(a) Greenpeace (b) merchants
(b) Climate Action Network (c) village landlords
(c) Climate Justice Alliance
(d) calligraphists
(d) Cross-Dependency Initiative

99. Consider the following statements regarding


96. Consider the following statements regarding
the Rights of Persons with Disabilities Act,
the Marrakech Partnership for Global Climate
2016:
Action:
1. It is a partnership between governments 1. It includes acid attack victims and

and the businesses and investors Parkinson's disease as disabilities.


launched at Sharm el-Sheikh Climate 2. The Parliament has been vested the
Change Conference (COP 27). power to add disabilities under the act.
2. It aims to immediately lower emissions 3. It increased the reservation for people
and increase resilience against climate with benchmark disabilities to 4% in
impacts. government jobs and 5% in higher
Which of the statements given above is/are education institutes.
correct? Which of the statements given above are
(a) 1 only
correct?
(b) 2 only
(a) 1 and 2 only
(c) Both 1 and 2
(b) 2 and 3 only
(d) Neither 1 nor 2
(c) 1 and 3 only
(d) 1, 2 and 3
97. Consider the following pairs:
Deccan Sultanates Dynasty
100. The terms 'High-Risk Jurisdictions subject to
1. Ahmadnagar : Nizam Shahi
a Call for Action' and 'Jurisdictions Under
2. Bijapur : Adil Shahi
Increased Monitoring' are often seen in the
3. Golkonda : Qutb Shahi
news in the context of:
4. Berar : Farooqi
(a) International Monetary Fund
How many pairs given above are correctly
matched? (b) INTERPOL

(a) Only one pair (c) Financial Action Task Force


(b) Only two pairs (d) World Bank
(c) Only three pairs
(d) All four pairs

Copyright © by Vision IAS


All rights are reserved. No part of this document may be reproduced, stored in a retrieval system or transmitted
in any form or by any means, electronic, mechanical, photocopying, recording or otherwise, without prior
permission of Vision IAS.
19 www.visionias.in ©Vision IAS
https://www.pdfnotes.co/

.
https://www.pdfnotes.co/

.
VISIONIAS
www.visionias.in
ANSWERS & EXPLANATIONS
GENERAL STUDIES (P) TEST – 3812 (2023)

Q 1.A
• When the Lok Sabha is dissolved, all business including bills, motions, resolutions, notices, petitions and
so on pending before it or its committee's lapse.
• They (to be pursued further) must be reintroduced in the newly-constituted Lok Sabha.
• However, some pending bills and all pending assurances that are to be examined by the Committee
on Government Assurances do not lapse on the dissolution of the Lok Sabha. Hence option 3 is not
correct.
• The position with respect to lapsing of bills is as follows:
o A bill pending in the Lok Sabha lapses (whether originating in the Lok Sabha or transmitted to it by the
Rajya Sabha).
o A bill passed by the Lok Sabha but pending in the Rajya Sabha lapses. Hence option 1 is correct.
o A bill not passed by the two Houses due to disagreement and if the president has notified the holding
of a joint sitting before the dissolution of Lok Sabha, does not lapse.
o A bill pending in the Rajya Sabha but not passed by the Lok Sabha does not lapse. Hence option
2 is not correct.
o A bill passed by both Houses but pending assent of the president does not lapse.
o A bill passed by both Houses but returned by the president for reconsideration of Houses does not lapse.

Q 2.C
• The Three Jewels (Triratna) embraced under Buddhism are :
o Buddha - The enlightened one
o Dhamma - Teachings of Buddha (doctrine)
o Sangha - The monastic order
• Out of the three jewels or triratnas, the concept of Sangha (the Order) was initiated by Buddha at the time
of giving the first sermon. These 5 companions along with Buddha became a Sangha (a group).
• According to Buddha, the Middle Path (Madhyam Marg) or Middle Way describes the character of the
Noble Eightfold Path that leads to liberation.
• Buddhism rejects the authenticity of the Vedas, i.e. it does not accept it. It also rejects the concept of
the existence of the eternal Soul (atman) unlike Jainism.
• Once Buddha attained Mahaparinirvana at Kushinagar in 483 BC, there was a need to compile his teachings,
and hence four Buddhist Councils were held in a span of the next 500 years to collate this material into
Pitakas.
• The result was the writing of three major pitakas Vinaya, Sutta, and Abhidhamma, that when
combined were called Tripitaka. All of these have been written in the Pali language. Hence option (c)
is the correct answer.
• Buddha attained Mahaparinirvana at Kushinagar (Malla Mahajanapada) in Uttar Pradesh at the age of 80 in
483 BC. He is said to be contemporary for a major part of his life to King Bimbisara and for the last few
years to Ajatashatru of the Haryanka Dynasty.
• Buddha is known in various Buddhist texts also as Tathagata and Sakyamuni. The predecessor of Buddha
under Buddhism was Kassapa Buddha and his successor will be Maitreya.

Q 3.A
• A Pigovian (Pigouvian) tax is a tax assessed against private individuals or businesses for engaging in
activities that create adverse side effects for society. Adverse side effects are those costs that are not included
as a part of the product's market price. These include environmental pollution, strains on public healthcare
1 www.visionias.in ©Vision IAS
https://www.pdfnotes.co/

.
from the sale of tobacco products, and any other side effects that have an external, negative impact. The
purpose of the Pigovian tax is to redistribute the cost back to the producer or user of the negative
externality. Hence option (a) is the correct answer.
• A carbon emissions tax or a tax on plastic bags are examples of Pigovian taxes. Negative externalities are
not necessarily “bad.” Instead, a negative externality occurs whenever an economic entity does not fully
internalize the costs of its activity. In these situations, society, including the environment, bears most of the
costs of economic activity. Thus Pigovian Tax is aimed to discourage negative production externalities in
the economy.

Q 4.C
• The Sea of Marmara, also known as the Marmara Sea, is an inland sea located entirely within the borders
of Turkey. It connects the Black Sea to the Aegean Sea via the Bosporus and Dardanelles straits, separating
the country's European and Asian sides.
• The Sea of Marmara, fabled for centuries for its blue waters and sparkling fish, laps the shores of Istanbul.
Its perfect form inspired a 19th-century historian to describe the ancient city as “a diamond set between two
sapphires.” But the Marmara has been sickening for a long time, and this year, it suffered a paroxysm that
choked its waters and suffocated marine life. In 2021 the shores of the Sea of Marmara were disfigured by
mucilage - nicknamed 'sea snot' - caused, at least in part, by the dumping of untreated waste into the
water. Hence, option (c) is the correct answer.

Q 5.A
• BIMSTEC stands for the Bay of Bengal Initiative for Multi-Sectoral Technical and Economic
Cooperation. BIMSTEC was established in 1997 with the aim of promoting economic and technical
cooperation among the member states. Its focus is on a wide range of areas, including trade and investment,
technology, energy, transport, agriculture, fisheries, tourism, and culture. BIMSTEC aims to create an
enabling environment for sustainable economic growth and development in the region. Hence statement 1
is correct.
• It is a regional organization comprising seven member states: Bangladesh, Bhutan, India, Myanmar,
Nepal, Sri Lanka, and Thailand. Hence statement 2 is not correct.
• The headquarters of BIMSTEC is located in Dhaka, the capital of Bangladesh.
• BIMSTEC members account for around 22 percent of the world's population and 4 percent of the world's
GDP (USD 3.8 trillion). Hence statement 3 is not correct.

Q 6.D
• Digital public goods are open-source software, open data, open AI models, open standards, and open
content that adhere to privacy and other applicable laws and best practices, do no harm by design
and help attain the Sustainable Development Goals (SDGs).
• The concept of digital public goods originally derives from the economic term “public good” which refers
to something that is non-excludable and non-rivalrous. Non-excludability indicates that you cannot prevent
someone from using or consuming the good. Non-rivalrousness indicates that one’s consumption or usage
does not limit or take away from someone else’s. The concept of public goods has since evolved into the
2 www.visionias.in ©Vision IAS
https://www.pdfnotes.co/

.
digital era. The digital environment is particularly conducive to public goods and has created new
methodologies for delivering public goods.
• India Stack (a set of APIs that allows governments, businesses, start-ups, and developers to utilize a unique
digital Infrastructure to solve India’s problems and to attain a presence-less, paperless, and cashless service
delivery model), Aadhar, UPI, and DigiLocker, eBooks, podcast and other tech-driven citizen-centric
services come under digital public good.
o DigiLocker- DigiLocker has 128 million registered users, 400 e-document types, and up to 5 billion e-
documents for consent-based sharing. The digital documents that can be stored on and retrieved from
DigiLocker include a PAN card, driving license, Aadhaar card, school mark sheets, and insurance
papers, among others. No other country provides a free service similar to DigiLocker to their citizens.
o Account aggregator is a gold standard in terms of giving user ownership of their data and the control to
use their data to access any digital services especially access to credit. This is a scaled and secure
implementation of the future of putting the customer in charge of their own data.
o Central KYC registry hosts more than 35 crores KYC records, with inter-usability across all financial
regulators — RBI, SEBI, IRDA, and the Pension Fund Regulatory and Development Authority
(PFRDA). This enables customers to interact with various financial entities nationwide without the need
to complete KYC formalities repeatedly.
• Hence option (d) is the correct answer.

Q 7.D
• The Provisions of the Panchayats (Extension to the Scheduled Areas) Act, 1996 (PESA) extends the
provisions of part IX of the constitution relating to the Panchayats to the Fifth Schedule areas. The
sixth schedule contains provisions related to the Administration of Tribal Areas. Hence, statement 2 is not
correct.
• At present (2019), ten states have Fifth Schedule Areas. These are: Andhra Pradesh, Telangana,
Chhattisgarh, Gujarat, Himachal Pradesh, Jharkhand, Madhya Pradesh, Maharashtra, Odisha and Rajasthan.
• The reservation of seats in the Scheduled Areas in every Panchayat shall be in proportion to the population
of the communities for whom reservation is sought to be given under Part IX of the Constitution. However,
the reservation for the Scheduled Tribes shall not be less than one-half of the total number of seats.
Further, all posts of Chairpersons of PRIs in the areas covered under PESA are reserved for tribal
communities and only persons belonging to tribal communities can contest for these Posts. Hence,
statement 1 is correct.
• As per PESA, every village shall have a Gram Sabha consisting of persons whose names are included in
the rolls for the Panchayat at the village level. PESA Act provides that the Gram Sabha shall be consulted
before making the acquisition of land for development projects. PESA empowers Gram Sabha to prevent
alienation of land in the Scheduled Areas and take appropriate action to restore unlawfully alienated land
of a Scheduled Tribe.
• Under PESA Act, it is mandatory to have the recommendation of Gram Sabha and Panchayats at the
appropriate level prior to the grant of a prospecting license or mining lease for minor minerals and
for the grant of concession for the exploitation of minor minerals by auction in the Scheduled
Areas. Hence, statement 3 is correct.

Q 8.D
• Open Radio Access Network, or Open RAN, is a key part of a mobile network system that uses cellular
radio connections to link individual devices to other parts of a network. It is frequently seen in the news in
the context of 5G technology.
• It comprises antennae, which transmits and receives signals to and from our smartphones or other
compatible devices. The signal is then digitised in the RAN-base station and connected to the
network. Hence statement 1 is correct.
• In the traditional set-up, Radio Access Network is provided as an integrated platform of both hardware and
software. Therefore, it is difficult to mix vendors for the radio and baseband unit, and in most cases, they
come from the same supplier.
• The idea of Open RAN is to change this, and enable operators to mix and match components.
• The Open RAN architecture allows for the separation - or disaggregation - between hardware and
software with open interfaces. Hence statement 2 is correct.
• RAN has been based on proprietary technologies of original equipment makers such as Ericsson, Nokia,
etc. With Open RAN, telecom players would have the flexibility to use in-house solutions or solutions from
multiple vendors for RAN services.

3 www.visionias.in ©Vision IAS


https://www.pdfnotes.co/

.
• This would allow telecom operators to look beyond traditional vendors, thus creating opportunities for
lesser-known vendors from abroad as well as from home to be part of the growing 5G ecosystem, based on
their innovation competence.
• Network flexibility is another advantage of the Open RAN architecture. Being software-centric, it is
scalable, agile and best of networks with improved network performance using artificial intelligence and
machine learning.
• Open RAN would reduce a telecom operator’s network deployment cost as it is interoperable with
other networks such as 4G. Hence statement 3 is correct.
• Open RAN is a new architecture, and not something that has been extensively tested. Therefore, there are
several challenges in the path to implement Open RAN such as latency issues, operations and maintenance.

Q 9.A
• Indian black honeybee (Apis karinjodian)
o A new endemic species of honeybee, the Indian black honeybee (Apis karinjodian), was
discovered in the Western Ghats. Its habitat ranges from the central Western Ghats and Nilgiris to the
southern Western Ghats, spanning the states of Goa, Karnataka, Kerala, and parts of Tamil Nadu. Hence
statement 1 is correct.
o The species has been classified as Near Threatened in the IUCN Red List. Hence statement 3 is not
correct.
o Interestingly, the new species has been found after more than 200 years as it was in 1798 the last honey
bee had been discovered in the country.
o The new species is found well for apiculture in India, as it has the ability to produce higher quantities
of honey which is thicker in consistency opening up new avenues for increasing honey production.
Hence statement 2 is correct.

Q 10.C
• Recently, the Union Cabinet, chaired by Honorable Prime Minister, approved the ratification of three
Protocols relating to amendments in the convention on International Civil Aviation (Chicago
Convention), 1944.
• The articles of the Chicago Convention establish the privileges and obligations of all contracting States and
promote international ICAO Standards and Recommended Practices (SARPs) adoption that regulate
international air transport.
• The Convention on International Civil Aviation, also known as the Chicago Convention, established
the International Civil Aviation Organization, a specialized agency of the United Nations charged
with coordinating international air travel. Hence option (c) is the correct answer.
• During the past 78 years, the Chicago Convention has undergone a few amendments and India has been
ratifying such amendments from time to time.

Q 11.B
• Parliamentary privileges can be classified into two broad categories: those that are enjoyed by each House
of Parliament collectively, and those that are enjoyed by the members individually.
• The privileges belonging to the members individually are:
• The Members of Parliament have freedom of speech in Parliament. No member is liable to any
proceedings in any court for anything said or any vote given by him in Parliament or its committees. This
freedom is subject to the provisions of the Constitution and to the rules and standing orders regulating the
procedure of Parliament. Hence, statement 1 is not correct.
• They cannot be arrested during the session of Parliament and 40 days before the beginning and 40 days
after the end of a session. This privilege is available only in civil cases and not in criminal cases or
preventive detention cases. Hence, statement 2 is correct.
• The members of Parliament are exempted from jury service. They can refuse to give evidence and appear
as a witness in a case pending in a court when Parliament is in session.

Q 12.D
• Technical textiles are defined as textile materials and products used primarily for their technical
performance and functional properties rather than their aesthetic or decorative characteristics. They are
manufactured using natural as well as man-made fibers.
• India’s strengths have already been defined in traditional textiles and natural fibers globally. It is the second
largest producer of polyester in the world and is now emerging as a key player in the technical textiles
industry contributing to a market size of $ 19 Bn. Technical textile is a fast-growing sub-segment that finds
4 www.visionias.in ©Vision IAS
https://www.pdfnotes.co/

.
its usage in an array of sectors. The end-use application of technical textiles is widespread and seen in
industries such as agriculture, construction, sports apparel, healthcare, etc. India’s leap towards
modernization and its manufacturing competitiveness are some of the key contributors to the growth of this
segment.
• Technical textile products exhibit enhanced performance over traditional textiles. Technical textile products
are manufactured using natural as well as man-made fibers such as Nomex, Kevlar, Spandex, Twaron, etc.
These fibers exhibit enhanced functional properties like higher tenacity, excellent insulation, improved
thermal resistance, etc. Hence, these fibers find application in varied industries and applications.

• Hence option (d) is the correct answer.

Q 13.D
• The McMohan Line runs from the tri-junction of India, Bhutan, and China to the tri-junction of India, China,
and Myanmar. It's an eastern sector of the international border between India and China along the state of
Arunachal Pradesh.
• Statement 1 is not correct: If you jump back to the Indian side, you will find yourself in Arunachal
Pradesh.
• Statement 2 is not correct: The latitude passing from your location will cut through Rajasthan but not
Nagaland.
• Statement 3 is correct: At the end of the line, there is a tri-junction of India, China, and Myanmar.

Q 14.A
• The fuel produced from combustible components that the industry calls Municipal Solid Waste is called
Refuse-Derived Fuel (RDF). It is a renewable energy source that ensures waste simply isn’t thrown into a
landfill and instead, put to good use. Hence statement 1 is not correct and 2 is correct.
• Generally, the waste treatment processes are classified into two categories: thermal (incineration,
gasification, and pyrolysis) and non-thermal (landfilling, aerobic, and anaerobic digestion).
o The landfill sites are becoming scarcer causing several environmental issues related to land pollution,
and underground water contamination. Incineration of solid wastes produces several carcinogenic
pollutants.
o Pyrolysis is an emerging technology for the efficient conversion and resource recovery of municipal
solid waste as it offers chemical and fuel value as well as char having several potential applications.
• Refuse-derived fuel (RDF) is a highly combustible fraction obtained from municipal solid waste
through mechanical biological treatment (MBT). MBT consists of magnets, trommels, shredders, and
eddy current separators which separate the recyclable components of MSW. RDF mainly consist of
plastics, yard waste, waste paper textiles, and very low amount of biomass components such as
kitchen waste. RDF contains a high amount of combustibles and a low amount of pollutants (chlorine,
mercury, cadmium) in comparison with municipal solid waste.
• State-of-the-art commercial applications of RDF for energy generation were found to be mainly by
cement industries for heating cement kilns but are also used for combined heat and power in
incinerators. It has found its application in co-firing biomass, coal, and poultry litter in conventional
waste-to-energy plants. It has no application as fuel in automobiles. Hence statement 3 is not correct.
5 www.visionias.in ©Vision IAS
https://www.pdfnotes.co/

.
Q 15.D
• India’s agricultural exports are poised to scale a new peak in the financial year ending March 31, 2023.
Government data show the value of farm exports in April-December 2022, at $39 billion, was 7.9% higher
than the $36.2 bn for the corresponding period of the previous year. At the present rate, the record $50.2 bn
exports achieved in 2021-22 look set to be surpassed.
• However, equally significant are the imports of agri produce, that at $27.8 bn in Apr-Dec 2022, have grown
15.4% over the $24.1 bn for Apr-Dec 2021.

• From the above India's agriculture imports along with exports have increased over the last decade. As a
result, there has been a further shrinking of the surplus on the farm trade account. The accompanying table
shows that the surpluses even in 2020-21 ($20.2 bn) and 2021-22 ($17.8 bn) were lower than the $22.7 bn
and $27.7 bn of 2012-13 and 2013-14 respectively. Hence both statements 1 and 2 are not correct.

Q 16.C
• Recently, 'Nucleoside Reverse Transcriptase Inhibitor (NRTI)' is news in the context of shortage of
medicines for people living with HIV.
• Based on its mechanism of action, antiretroviral drugs for HIV treatment are categorized into the following
different classes i.e.,
o viral entry blockers (the CCR5 receptors antagonists)
o non-nucleoside and nucleoside RT inhibitors (NNRTIs and NRTIs, respectively);
o integrase strand transfer inhibitors and
o protease inhibitors.
• Nucleoside reverse-transcriptase inhibitors (NRTIs) are a class of antiretroviral drugs used to treat HIV
infection or AIDS, and in some cases hepatitis B. RTIs inhibit activity of reverse transcriptase, a viral
DNA polymerase that is required for replication of HIV and other retroviruses. Hence option (c) is the
correct answer.

Q 17.C
• The magnificent Kailashanatha temple at Ellora, built during the reign of Rashtrakuta king Krishna
I, is the culmination of at least a millennium-long tradition in rock-cut architecture in India.
o It is a complete dravida building with a Nandi shrine—since the temple is dedicated to Shiva—
a gopuram-like gateway, surrounding cloisters, subsidiary shrines, staircases, and an imposing
tower, or vimana, rising to thirty metres.
o Importantly, all of this is carved out of living rock. One portion of the monolithic hill was carved
patiently to build the Kailashnath temple. Hence statement 1 is not correct.
• Practically all the surfaces of the temple complex are richly ornamented with bold, dramatic, and
exceptionally fine sculptures.
o Most of them are Shaiva, but there are also representations of Vishnu.
o In fact, the sculptures to the left of the entrance are mostly Shaiva, while those to the right are mostly
Vaishnava.
o A similar logic of sculptural arrangement is found along the back wall of the gallery surrounding the
temple.
6 www.visionias.in ©Vision IAS
https://www.pdfnotes.co/

.
o The sculptures include representations of Shiva, Shiva and Parvati, Durga, the Sapta-Matrikas,
Ganesha, and the goddesses Ganga, Yamuna, and Sarasvati. Hence statement 2 is correct.
• The Ellora (also known as Elura or Elapura) Caves are one of the largest rock-cut monastery cave complexes
in the world.
o There are around 100 Buddhist, Hindu and Jain caves at the site, of which 34 are open to the public.
They are locally known as Verul Leni.
o Of these 34 caves in the complex, cave number 16, popularly known as Kailash or Kailashnath temple
and dedicated to Lord Shiva, is one of the largest solitary monolithic stone excavations in the world.
o The temple was carved out vertically from top to bottom from a single basalt rock. It is protected by
the Archeological Survey of India and was also declared a UNESCO World Heritage Site in
1983. Hence statement 3 is correct.

Q 18.D
• According to the National Oceanic and Atmospheric Administration (NOAA), subsidence is the “sinking
of the ground because of underground material movement”. It can happen for a host of reasons, man-
made or natural, such as the removal of water, oil, or natural resources, along with mining activities.
• Earthquakes, soil erosion, and soil compaction are also some of the well-known causes of subsidence.
• The exact reason behind Joshimath land subsidence is still unknown but experts suggest that the incident
might have occurred because of unplanned construction, over-population, obstruction of the natural
flow of water and hydel power activities. Not only this, the area is a seismic zone, which makes it prone
to frequent earthquakes. Apart from the aforementioned possible reasons, reports have pointed out that
subsidence in Joshimath might have been triggered by the reactivation of a geographic fault — defined
as a fracture or zone of fractures between two blocks of rock — where the Indian Plate has pushed under
the Eurasian Plate along the Himalayas.
• Hence, option (d) is the correct answer.

Q 19.B
• Recent Context: The Employees' Provident Fund Organisation (EPFO) has introduced new guidelines,
under which subscribers and employers can jointly apply for higher pensions under the Employees' Pension
Scheme.
• The EPFO is a statutory body formed by the Employees' Provident Fund and Miscellaneous
Provisions Act, 1952.
• It is under the administrative control of the Ministry of Labour and Employment, Government of
India. Hence statement 3 is not correct.
• EPFO manages social security funds of workers in the organized/semi-organized sector in India.
Hence statement 1 is correct.
• For the portability of Provident Fund accounts, EPFO has a “Universal Account Number” Service for
contributing members. Hence statement 2 is correct.
• It operates three schemes: Employee’s Provident Fund Scheme, 1952, Pension Scheme 1995 (EPS) and
Insurance Scheme 1976.

Q 20.C
• The Anglo-Mysore Wars were a series of wars fought during the last three decades of the 18th century
between the Kingdom of Mysore on the one hand, and the British East India Company (represented chiefly
by the Madras Presidency), Maratha Empire, Kingdom of Travancore and the Nizam of Hyderabad on the
other. Haider Ali and his successor Tipu Sultan fought a war on four fronts with the British attacking from
the west, south and east, while the Nizam's forces attacked from the north.
• First Anglo-Mysore war (1767-69): The British, after their success in the Battle of Buxar with the
nawab of Bengal, signed a treaty with the Nizam of Hyderabad persuading him to give them the
Northern Circars for protecting the Nizam from Haidar Ali who already had disputes with the
Marathas. The Nizam of Hyderabad, the Marathas, and the English allied together against Haidar
Ali. Haidar changed his strategy and suddenly appeared before the gates of Madras causing complete
chaos and panic at Madras. This forced the English to conclude a treaty with Haidar on April 4, 1769
known as the Treaty of Madras. Hence statement 2 is correct.
• The Second Mysore War (1780-84) came to an end with the Treaty of Mangalore in 1783. Accordingly,
all conquests were mutually restored and the prisoners on both sides were liberated.
• Third Anglo Mysore War (1790-92): The Treaty of Srirangapattinam was signed after the third
war. According to the provisions of this treaty, Tipu had to give up half his dominions, he had to pay a
war indemnity of three crore rupees and surrender two of his sons as hostages to the English and both sides
7 www.visionias.in ©Vision IAS
https://www.pdfnotes.co/

.
agreed to release the prisoners of war. Thus, the state of Mysore was not abolished after this treaty.
Hence, statement 1 is not correct.
o The rule of Tipu came to an end after the fourth Anglo-Mysore war and the English restored
Wodeyar's rule at Mysore.
• The circumstances which led to the Fourth Mysore War can be summarized as: Tipu Sultan wanted to
avenge his humiliating defeat and the terms imposed on him by the British. He took efforts to seek the help
of France, Arabia, Kabul and Turkey. He corresponded with the Revolutionary French Government in July
1798. At Srirangapattinam, a Jacobian Club was started and the flag of the French Republic was hoisted.
The tree of Liberty was also planted. Later, when Napoleon came to power, Tipu received a friendly letter
from Napoleon (who was in Egypt at that time).
• It was at this juncture that Wellesley with a mind already filled with fear of Napoleon prepared for a war
against Mysore.
• Lord Cornwallis succeeded Warren Hastings as Governor-General in 1786. Thus, he came after the first
Anglo-Mysore War (1767-69). He led the English forces in the Third Anglo-Mysore War (1790-92).
Hence, statement 3 is not correct.

Q 21.B
• In December 1927, a large number of Muslim leaders had met at Delhi at the Muslim League session and
evolved four proposals for their demands to be incorporated into the draft constitution (Nehru Report).
These proposals, which were accepted by the Madras session of the Congress (December 1927), came to
be known as the ‘Delhi Proposals’. These were:
o joint electorates in place of separate electorates with reserved seats for Muslims;
o one-third representation to Muslims in Central Legislative Assembly. Hence, option (b) is the
correct answer.
o representation to Muslims in Punjab and Bengal in proportion to their population;
o formation of three new Muslim majority provinces— Sindh, Baluchistan and North-West
Frontier Province.

Q 22.A
• Financial intermediaries in the economy deal extensively with borrowing and lending, and the net interest
margin is the net benefit of lending.
• Net interest margin or NIM denotes the difference between the interest income earned and the interest paid
by a bank or financial institution relative to its interest-earning assets. Hence statement 1 is correct.
• It is an industry-specific profitability ratio for banks and other financial institutions that lend out interest-
earning assets. If deposit rates outpace lending rates, then bank NIMs will shrink.
• In case the demand for savings increases relative to the demand for loans, the NIM will fall. An increase in
savings and a decline in demand for loans would eventually lead to a decline in interest rates in the economy.
Also, NIM falls as more savings and lesser loans would result in low investment returns. Hence statement
2 is not correct.
• A higher NIM would increase the profitability of the lender. A negative NIM indicates that the lender has
been unable to make good use of its assets, as returns produced by investments have failed to offset interest
expenses. Thus, NIM is a significant indicator of the financial stability of a lender.

Q 23.C
• DNA fingerprinting is a laboratory technique used to determine the probable identity of a person based on
the nucleotide sequences of certain regions of human DNA that are unique to individuals. DNA
fingerprinting is used in a variety of situations, such as criminal investigations, other forensic purposes
and paternity testing.
• The technique was developed in 1984 by British geneticist Alec Jeffreys, after he noticed that certain
sequences of highly variable DNA (known as minisatellites), which do not contribute to the functions of
genes, are repeated within genes.
• The DNA fingerprinting started with Restriction Fragment Length Polymorphisms (RFLP), which
was tedious and time consuming but opened the doors for new developments in the arena. Now,
the markers being used are short tandem repeats (STRs), species-specific primers, SNPs (single
nucleotide polymorphism), NGS (next-generation sequencing), Y-STR, X-STR, and mitochondrial
DNA (mtDNA).
o A minisatellite is a tract of repetitive DNA in which certain DNA motifs (ranging in length from 10
to 60 base pairs) are typically repeated 5–50 times. Minisatellites are notable for their high mutation
rate and high diversity in the population.
8 www.visionias.in ©Vision IAS
https://www.pdfnotes.co/

.
o Restriction fragment length polymorphism (RFLP) is a technique that exploits variations in
homologous DNA sequences, known as polymorphisms, in order to distinguish individuals,
populations, or species or to pinpoint the locations of genes within a sequence.
• Hence option (c) is the correct answer.

Q 24.D
• The revolt of 1857 began on 10 May 1857 when the sepoys in the cantonment of Meerut broke out in
mutiny. It began in the lines of the native infantry, spread very swiftly to the cavalry and then to the city.
• Before the Meerut incident, the 19th Native Infantry at Berhampur, which refused to use the newly
introduced Enfield rifle and broke out in mutiny in February 1857 was disbanded in March 1857.
• On May 11, 1857: The sepoys arrived at the gates of the Red Fort and declared the Mughal Emperor
Bahadur Shah II was proclaimed the Emperor of India.
• The Azamgarh Proclamation was issued by the rebels on 25 August 1857. It is one of the main sources
of our knowledge about what the rebels wanted. It had denouncements of the tyranny and oppression of the
English people. It's a democratic and secular document that aims to unite both Hindus and Muslims against
the tyranny of the East India company.
• Very soon the rebellion spread throughout Northern and Central India at Lucknow, Allahabad, Kanpur,
Bareilly, and Banaras in parts of Bihar, Jhansi and other places.
• Kanpur was lost to the British on 5th June 1857. Nana Sahib was proclaimed the Peshwa. At Kanpur,
Nana Sahib was joined by his able and experienced Lieutenant, Tantia Tope.
• The British captured Delhi on September 20, 1857 after prolonged and bitter fighting. John Nicholson,
the leader of the siege, was badly wounded and later succumbed to his injuries. Bahadur Shah was taken
prisoner. The royal princes were captured and butchered on the spot, publicly shot at point blank range by
Lieutenant Hudson himself. The emperor was exiled to Rangoon where he died in 1862.
• Hence option (d) is the correct answer.

Q 25.D
• The Ministry of Parliamentary Affairs organises the All India Whips Conference, at the Centre as
well as in the States. The Whips’ Conference usually makes a number of recommendations for the smooth
and efficient working of Parliament and the State Legislatures in light of the experience gained by the
Whips. Hence statement 1 is correct.
• The office of ‘whip’, is mentioned neither in the Constitution of India nor in the Rules of the House
nor in a Parliamentary Statute. It is based on the conventions of the parliamentary government. Every
political party, whether ruling or Opposition has its own whip in the Parliament.
• Government Chief Whip has some very important duties. The most important duty is mapping out the time
of the session, coordinating, monitoring and management of the business of the Government. The
Government whips also act as an important communication link between the leader of the House and
the Members of the ruling party and also keep in close touch with the whips of the other parties on matters
concerning the business of the House as also on many other matters relating to the House as a whole. Hence
statement 2 is correct.
• The whip of the party is appointed by the political party to serve as an assistant floor leader. He is
charged with the responsibility of ensuring the attendance of his party members in large numbers and
securing their support in favour of or against a particular issue. He regulates and monitors their behaviour
in the Parliament. The members are supposed to follow the directives given by the whip. Hence
statement 3 is correct.

Q 26.B
• Recently, France—alongside several EU states and Canada—announced its withdrawal of troops and
military resources from Mali. This development will primarily affect Operation Barkhane, a French-
led counter-insurgency operation (and the largest external counterterrorism force) in the Sahel.
Hence option (b) is the correct answer.
• Operation Barkhane, a counter-terrorism operation in the Sahel region, was launched in August 2014
by France. The operation was authorized by the United Nations Security Council (UNSC).
• France began its military operations in the Sahel in January 2013. Titled Operation Serval, it was limited
to targeting Islamic extremists linked to al-Qaeda who took control of northern Mali. However, in 2014, the
mission was scaled up, renamed Operation Barkhane and was aimed at counter-terrorism. The objective
was to assist local armed forces to prevent the resurgence of a non-state armed group. Operation Barkhane
aimed at counter-terrorism in Mali, Niger, Burkina Faso, Mauritania and Chad and not all the Sahel region.

9 www.visionias.in ©Vision IAS


https://www.pdfnotes.co/

.
• The Sahel, meaning “the shore” in Arabic, is a vast area crossing 6,000 kilometers from East to West Africa.
It covers many geographic and agroecological systems, 12 countries and is home to 400 million people.
• The political region of the Sahel, as defined by the United Nations strategy (UNISS), covers 10
countries (Senegal, Gambia, Mauritania, Guinea, Mali, Burkina Faso, Niger, Chad, Cameroon and
Nigeria). The region faces many challenges. Climate change threatens to further degrade land, vegetation,
water resources and food systems through increased incidence of drought, desertification and floods and
projected shortening of the rainy season. The Sahel ecological zone has shifted from 50 to 200 kilometers
southward over the last three decades, resulting in biodiversity and arable land losses.

Q 27.D
• There are three main types of unemployment – cyclical, structural, and frictional unemployment. In practice,
these cannot be measured directly, and they can often overlap, but they provide a useful way of thinking
about unemployment.
• Frictional unemployment: Frictional unemployment occurs when people move between jobs in the
labor market, as well as when people transition into and out of the labor force. This type of
unemployment is generally shorter term (less than one month). Frictional unemployment is likely to occur
at all points of the business cycle and, like structural unemployment, may not influence wages or inflation.
• Structural unemployment: Structural unemployment is caused by fundamental shifts in the economy
that make it difficult for workers to find employment. Structural unemployment occurs when there is a
mismatch between the jobs that are available and the people looking for work. This mismatch could be
because job seekers don’t have the skills required to do the available jobs, or because the available jobs are
a long way from the job seekers. Hence statement 1 is not correct.
o Structural unemployment tends to be longer lasting than other types of unemployment. This is
because it can take a number of years for workers to develop new skills or move to a different region to
find a job that matches their skills. As a result, workers who are unemployed because of structural
factors are more likely to face long-term unemployment (for more than 12 months). In contrast to
cyclical unemployment, structural unemployment exists even when economic conditions are good.
• Seasonal unemployment: Seasonal unemployment occurs when people are unemployed at particular times
of the year when demand for labor is lower than usual. For example in sectors such as agriculture and
tourism, production takes place only in certain seasons.
• Cyclical Unemployment: Cyclical unemployment occurs with changes in economic activity over the
business cycle. During an economic downturn, a shortfall of demand for goods and services results in a lack
of jobs available for those who want to work. Businesses experiencing weaker demand might reduce the
number of people they employ by laying off existing workers or hiring fewer new workers. As a result,
people looking for work will also find it harder to become employed. The opposite situation occurs when
demand strengthens.
o Hence recent layoffs by tech companies can lead to an increase in cyclical unemployment. Hence
statement 2 is not correct.

Q 28.B
• Bor Thekera (Garcinia pedunculata) is a medicinal plant in Assamese found to have cardioprotective
potential.
o It is an evergreen tree endemic to South Eastern Asia.

10 www.visionias.in ©Vision IAS


https://www.pdfnotes.co/

.
o Traditionally it is forbidden for raw consumption.
o Sun-dried slices of ripe fruit are used for culinary and medicinal purposes and are known to have
therapeutic properties like anti-inflammatory, antibacterial, antidiabetic, hypolipidemic,
nephroprotective, and even neuroprotective activity.
o It is also a rich source of antioxidants.
• Hence option (b) is the correct answer.

Q 29.B
• A new line of political activity to keep up the spirit of resistance to colonial rule was advocated by C.R. Das
and Motilal Nehru. They suggested that the nationalists should end the boycott of the legislative councils,
enter them, expose them as 'sham parliaments and as a mask which the bureaucracy has put on,' and obstruct
'every work of the council.' This, they argued, would not be giving up non-cooperation but continuing it in
a more effective form by extending it to the councils themselves. It would be opening a new front in the
battle.
• C.R. Das as the President of the Congress and Motilal as its Secretary put forward this programme
of either mending or ending' the councils at the Gaya session of the Congress in December
1922. Another section of the Congress, headed by Vallabhbhai Patel, Rajendra Prasad and C.
Rajagopalachari, opposed the new proposal which was consequently defeated by 1748 to 890 votes.
• Das and Motilal resigned from their respective offices in the Congress and on 1 January 1923
announced the formation of the Congress-Khilafat Swaraj Party better known later as the Swaraj
Party. Das was the President and Motilal one of the Secretaries of the new party. Hence statement 1 is not
correct.
• The adherents of the council-entry programme came to be popularly known as 'pro-changers'and those
still advocating boycott of the councils as 'no-changers.'
• Very soon, a group of Responsivists arose in the Swarajist party who wanted to work the reforms and
to hold office wherever possible. The Responsivists joined the Government in the Central Provinces.
Their ranks were soon swelled by N.C. Kelkar, M.R. Jayakar and other leaders. Lajpat Rai and Madan
Mohan Malaviya too separated themselves from the Swaraj Party on Responsivist as well as communal
grounds. Hence statement 2 is not correct.
• On 6 November 1924, Gandhiji brought the strife between the Swarajists and no-changers to an
end, by signing a joint statement with Das and Motilal that the Swarajist Party would carry on work in the
legislatures on behalf of the Congress and as an integral part of the Congress. This decision was endorsed
in December at the Belgaum session of the Congress over which Gandhiji presided. He also gave the
Swarajists a majority of seats on his Working Committee. Hence statement 3 is correct.

Q 30.B
• Global Carbon Budget
o The Global Carbon Budget is produced by 76 scientists from 57 research institutions in 15 countries
working under the umbrella of the Global Carbon Project (GCP). Hence statement 1 is not correct.
o The budget provides an in-depth look at the amount of fossil fuels that nations around the world burn
and where it ends up.
• About Global Carbon Project:
o The Global Carbon Project (GCP) integrates knowledge of greenhouse gases for human activities
and the Earth system. Our projects include global budgets for three dominant greenhouse gases
— carbon dioxide, methane, and nitrous oxide — and complementary efforts in urban, regional,
cumulative, and negative emissions.
o It is a Global Research Project of Future Earth and a research partner of the World Climate
Research Programme.
o It was established in 2001 by a shared partnership between the International Geosphere-Biosphere
Programme (IGBP), the International Human Dimensions Programme on Global Environmental
Change (IHDP), the World Climate Research Programme (WCRP) and Diversitas. This partnership
constituted the Earth Systems Science Partnership (ESSP) which subsequently evolved into Future
Earth.
• Global Carbon Budget Report 2022
o Preliminary data for 2022 show an increase in fossil CO2 emissions relative to 2021 of +1.0% (range
0.1% to 1.9%) globally, primarily driven by growth in oil use from the delayed rebound of aviation
since the COVID-19 pandemic.
o 2020 emissions declined 5.4% from 2019 levels because of COVID-19 measures.

11 www.visionias.in ©Vision IAS


https://www.pdfnotes.co/

.
o The latest data confirm that the rate of increase in fossil CO2 emissions has slowed, from 3% per
year during the 2000s to about +0.5% per year in the past decade. Hence statement 2 is correct.

Q 31.A
• A semiconducting material that is composed only of a single type of atom, such as a silicon atom, is known
as a pure or intrinsic semiconductor. Impurities are added into a semiconductor to actually increase the
electric conductivity. Hence option (a) is the correct answer.
• The process of adding an impurity into the semiconductor to increase its ability to conduct electricity is
known as doping and the impure semiconductor is known as a doped semiconductor. Pure Silicon or
Germanium are rarely used as semiconductors. Practically usable semiconductors must have controlled
quantity of impurities added to them. Addition of impurity will change the conductor ability and it acts as
a semiconductor.
• There are two types of doped semiconductors - the n-type semiconductor and the p-type semiconductor.
When we add an impure atom that contains more valence electrons than the original atom of the
semiconductor, such a semiconductor is known as an n-type doped semiconductor. However, when we add
an atom that has less valence electrons, we call this type of semiconductor the p-type doped semiconductor.

Q 32.C
• Recent Context: Ministry of Statistics and Programme Implementation (MoSPI) released revised
guidelines to broaden the scope of the Member of Parliament Local Area Development Scheme (MPLADS)
so as to enable MPs to recommend developmental works as per changing needs of the community.
• Also, a new Web Portal was launched for the implementation of the Revised Fund Flow Procedure under
MPLADS. Hence statement 1 is correct.
o It will facilitate real-time monitoring, greater transparency, and accountability in the system.
• RECOMMENDATION OF WORK BY MPs UNDER MPLADS
o Lok Sabha MPs can recommend works in their Lok Sabha constituencies.
o Rajya Sabha MPs can recommend within the state of the election.
o Nominated members of both Houses can recommend working anywhere in the country. Hence
statement 2 is correct.
• About MPLADS:
o MPLADS, a Central Sector Scheme launched in 1993, enables MPs to suggest and get executed
developmental works of a capital nature based on locally felt needs with an emphasis on the creation of
durable assets.
o Under it, MPs receive Rs.5 crore each year in two installments of Rs. 2.5 crore each.
o Every year, M.P. shall recommend at least 15% of MPLADS funds for areas inhabited by the SC
population and 7.5 percent for areas inhabited by the ST population.
o Funds under MPLADS are non-lapsable.
o District Authority is responsible for the overall coordination and supervision of works under the scheme
at the district level.
o MoSPI is responsible for policy formulation, the release of funds, and prescribing monitoring
mechanisms for the implementation of the Scheme.
Q 33.A
• Article 136 of the Indian Constitution provides for Special leave to appeal by the Supreme Court.
The Supreme Court is authorised to grant in its discretion special leave to appeal from any judgement
in any matter passed by any court or tribunal in the country (except military tribunal and court-
martial).
• Characteristics of Special leave to appeal:
o It is a discretionary power and hence, cannot be claimed as a matter of right. Hence, statement 1 is
correct.
o It can be granted in any judgement whether final or interlocutory. Hence, statement 2 is not
correct.
o It may be related to any matter–constitutional, civil, criminal, income-tax, labour, revenue,
advocates, etc. Hence, statement 3 is not correct.
o It can be granted against any court or tribunal and not necessarily against a high court (of course, except
a military court).
• Article 136 is divided into two stages:
o The court, while hearing the petition for special appeal to leave, will see whether the petitioner should
be granted such leave or not. The court is exercising its discretionary power to grant or not grant the
appeal to leave.
12 www.visionias.in ©Vision IAS
https://www.pdfnotes.co/

.
o If the petition is granted, the special leave petition will be converted into an appeal, so the court will
exercise its appellate jurisdiction to decide the case.

Q 34.A
• Sealed cover jurisprudence is the practice followed by the Supreme Court (and sometimes lower courts
as well) of seeking and accepting information from government agencies in sealed envelopes that can only
be perused by the judges. A court can ask for information in a sealed cover in broadly two circumstances:
when information is connected to an ongoing investigation, and when it involves personal or confidential
information.
• However the practice prevents parties from having a full overview of the charges against them, and it
is not compatible with the idea of an open court and a transparent system of justice. It also takes away the
opportunity to analyze judicial decisions, and to appreciate the rationale behind them. Hence, statement 1
is correct.
• Rule 7 of Order XIII of the Supreme Court Rules, 2013 provides that the Chief Justice or the court
can, through a judicial order, direct any document to be kept confidential in a sealed cover if
publication of the records is “considered to be not in the interest of the public”. So, besides the Chief Justice
of India, the Supreme court also has powers to direct any document to be kept confidential in a sealed cover.
Hence, statement 2 is not correct.

Q 35.A
• Recent context: The United Nations Climate Change Conference COP27 closed today with a
breakthrough agreement to provide “loss and damage” funding for vulnerable countries hit hard by
climate disasters.
• The establishment of a Loss and Damage Fund was, for many, the highlight of the United Nations Climate
Conference (COP 27) and the culmination of decades of pressure from climate-vulnerable developing
countries. The fund aims to provide financial assistance to nations most vulnerable and impacted by
the effects of climate change. Hence, statement 1 is correct.
• One of the big letdowns has been the lack of any progress on increasing the financial flows for addressing
climate change. The agreement takes note of the fact that about USD 4 trillion are required every year as
investments in the renewable energy sector till 2030 to reach net-zero targets. And yet, the developed
countries have not yet delivered on their promise to deliver even the relatively small amount of USD 100
billion every year. There is nothing in the agreement that puts pressure on the developed countries to
deliver these, and more, at the earliest. At COP15 in 2009, developed countries committed to a collective
goal of mobilising USD 100 billion per year by 2020 to support climate action in developing countries.
Hence, statement 2 is not correct.

Q 36.C
• The Mayor of the Municipal Corporation of Delhi (MCD) is elected by the Councillors of the
Municipal Corporation of Delhi. The party that bags the highest number of seats in the MCD elections
proposes a name for the post. If the other party also proposes a name, elections are held. The elected
councillors take part in the voting process. Hence option (c) is the correct answer.
• In addition to the councillors, MLAs, and Lok Sabha and Rajya Sabha MPs from Delhi are also eligible to
vote in the election this year. Nominated members do not usually cast votes. The polling takes place through
a secret ballot.
• Duties of the Mayor: The mayor is responsible for convening a special meeting at the corporation if at least
one-fourth of members express their willingness for it.
• Moreover, it can suspend the meeting if a quorum of more than one-fifth of the total councillors is not
reached.

Q 37.A
• The Standard Model (SM) is the theory that describes the properties of all subatomic particles, classifies
them into different groups, and determines how they’re affected by three of the four fundamental forces of
nature: strong-nuclear force, weak-nuclear force, and the electromagnetic force (it can’t explain
gravity).
• It explains how particles called quarks (which make up protons and neutrons) and leptons (which include
electrons) make up all known matter. It also explains how force carrying particles, which belong to a
broader group of bosons, influence the quarks and leptons. Hence statement 1 is correct.
• The Standard Model can’t explain why the universe has more matter than antimatter, what dark
matter is, or what dark energy is. Dark matter candidates arise frequently in theories that suggest physics
13 www.visionias.in ©Vision IAS
https://www.pdfnotes.co/

.
beyond the Standard Model, such as supersymmetry and extra dimensions. Hence statement 3 is not
correct.
• General relativity is Albert Einstein's understanding of how gravity affects the fabric of space-
time. Special relativity argued that space and time are inextricably connected. Whereas Standard Model
describes the other three fundamental forces. Both theories have different spheres of influence. Uniting
them is one of the big challenges of physics. Hence statement 2 is not correct.

Q 38.B
• Muhammad bin Tugluq was deeply read in religion and philosophy and had a critical open mind. He not
only conversed with Muslim mystics but also with Hindu yogis and Jain scholars. He honoured
Jinaprabha Suri, one of the esteemed Jain saints.
• The historical record says that Jinaprabha Suri visited the court of Muhammad-bin-Tughlaq of Delhi in
1328. The king treated him with respect, seated him by his side and a procession was arranged in his honour,
escorted by Maliks.
• Hence option (b) is the correct answer.

Q 39.D
• The National Industrial Corridor Programme (NICP) has been conceived to promote world-class
manufacturing facilities and develop futuristic industrial cities in India. The Delhi-Mumbai Industrial
Corridor was the first corridor taken up for development in 2011.
• The broad objective of NICP is to provide plug-and-play infrastructural facilities for setting up large-scale
manufacturing units. It is being implemented in close cooperation with the State Governments who provide
land as their share of equity in the programme.
• The National Industrial Corridor Development and Implementation Trust (NICDIT) under the
DPIIT is the overarching institutional framework to implement the programme. Hence statement 3
is not correct.
• Under the overall framework of the National Master Plan for multimodal connectivity of the Industrial
Corridor Programme, it is proposed to implement the following 11 Industrial Corridors projects. Hence
statement 1 is not correct.
o Delhi-Mumbai Industrial Corridor (DMIC)
o Amritsar-Kolkata Industrial Corridor (AKIC)
o Chennai-Bengaluru Industrial Corridor (CBIC)
o Vizag-Chennai Industrial Corridor (VCIC)
o Extension of CBIC to Kochi via Coimbatore
o East Coast Industrial Corridor (ECIC)
o Hyderabad-Nagpur Industrial Corridor (HNIC)
o Hyderabad-Warangal Industrial Corridor (HWIC)
o Hyderabad-Bengaluru Industrial Corridor (HBIC)
o Bengaluru Mumbai Industrial Corridor (BMIC)
o Delhi-Nagpur Industrial Corridor (DNIC)
• 32 Nodes have been identified for development in the 11 Industrial Corridors which would be developed in
4 Phases based on the availability of land, preparedness and multi-modal connectivity infrastructure being
provided by the different line Ministries under the various flagship schemes of the Government of India.
• Recently, Prime Minister has laid the foundation stone of South India’s 1st Industrial Corridor Project to be
implemented at Tumakuru which is spread over 8500 Acre of land under the Chennai Bengaluru Industrial
Corridor. Hence statement 2 is not correct.
• The Government of India, through the National Industrial Corridor Development and Implementation Trust
(NICDIT), and the Government of Karnataka, through the Karnataka Industrial Area Development Board
(KIADB), have taken up the development of the Industrial Township at Vasanthanarsapura, spread across
8500 acres in three phases in Tumakuru district through the project Special Purpose Vehicle (SPV).
• The CBIC project includes the development of industrial townships at three nodes: Krishnapatnam (Andhra
Pradesh), Tumakuru (Karnataka), and Ponneri (Tamil Nadu). The Tumakuru node has been prioritized for
development, with an area of 1736.20 acre being identified as the priority development area

Q 40.D
• The committee of secretaries (COS) has been taking care of coordination activities in the central secretariat.
The Cabinet Secretary in Union government is the senior-most civil servant of the country, but his functions
and roles have little to do with the Union ministries. He leads his cabinet secretariat which is a special and
distinct organisation, separate from the central secretariat.
14 www.visionias.in ©Vision IAS
https://www.pdfnotes.co/

.
• Hence it can be maintained that there is no office in the Union government which can be equated to that of
the chief secretary in the state. To some extent, the Cabinet Secretary can be called the counterpart of the
chief secretary but the functions performed and the roles assumed by the chief secretary in the state
administration are so vast, varied and wide that they are shared at the Central level, by the Cabinet Secretary,
the personnel secretary, the home secretary and the finance secretary.
• Still, the comparison can be attempted on the basis of similarities.
o Both functionaries are chief advisors to their respective chief executives.
o Both of them are chief coordinators of their respective administrations.
o Both are secretaries to their respective cabinets at both levels.
o Both are administrative heads of their respective cabinet secretariats.
o Both offices originated at the Central level because of their functional relevance.
o Both supervise the implementation of the decisions of their respective cabinets.
o Both are heads of their respective civil services.
• The differences between the two are genuine and deep:
o The powers and functions of the chief secretary are much more than that of the Cabinet Secretary.
o The former is the administrative head of the state secretariat, while the latter Is not the admin-
istrative head of the central secretariat. Hence statement 1 is not correct.
o The chief secretary is the chief of state secretaries, while the Cabinet Secretary is not the chief of
central secretaries but only a primus Interprets or first among equals. Hence statement 2 is not
correct.
o The chief secretary is the Residual Legatee at the state level, while the Cabinet Secretary does not
perform this legal function at the Centre. At the central level, It Is performed by the Principal Secretary
to the Prime Minister who Is the administrative head of the PM office. Some departments of the state
secretariat are under the direct charge of the chief secretary who does not head, the state cabinet
secretariat.

Q 41.C
• Hypermetropia is also known as far-sightedness.
• A person with hypermetropia can see distant objects clearly but cannot see nearby objects distinctly.
Hence statement 1 is not correct.
• The near point, for the person, is farther away from the normal near point (25 cm).
• Such a person has to keep reading material much beyond 25 cm from the eye for comfortable reading.
• This is because the light rays from a closeby object are focused at a point behind the retina.
• This defect arises either because ---
o The focal length of the eye lens is too long, or
o The eyeball has become too small. Hence statement 2 is not correct.
• This defect can be corrected by using a convex lens of appropriate power. Hence statement 3 is
correct.
• Eyeglasses with converging lenses provide the additional focusing power required for forming the image
on the retina.

15 www.visionias.in ©Vision IAS


https://www.pdfnotes.co/

.
Q 42.C
• Definition of ‘Forest’ (as defined in the Indian forest act, 1927) -an area occupied by the Government
for conservation and management of biological and ecological resources is called a forest.
• Types of Forest Land under the Indian Forest Act, 1927:
o Private Forests. The private forests are those which are not occupied by the Government.
o According to the Indian Forests Act, 1927, the Government is commanded to regulate timber
cutting and cultivation, etc. Private forest land also allows the state government to capture the
forest land for public purposes. Hence statement 1 is correct.
o Public Forests: Public forests are of three types:
✓ Reserved Forests: These are the forests that are reserved by the Government. More than half of
the forest area in India is declared as a Reserve forest. 53% of the total forests in India are Reserved
Forests. The activities like hunting and cutting trees are strictly banned in these forests. Only with
special permission from the higher authorities, these activities may be performed.
✓ Village Forests: According to the Indian Forests Act, 1927, the State Government can give the
rights to any village group, which the Government has over any land which has been
reserved. Rules for regulating the management of the forest to be made by the State
Government. Hence statement 2 is correct.
✓ Protected Forests: The Indian Forest Act empowers the State Government to use any land as
protected forest. It must be noted that these forests are not reserved under the state government. A
protected forest can be a reserved forest but a reserved forest cannot be a protected forest.

Q 43.D
• Eastern Indian temples include those found in the North East, Bengal and Odisha. Each of these three areas
produced distinct types of temples.
• The temples of Odisha constitute a distinct sub-style within the nagara order called Kalinga style.
o Most of the main temple sites are located in ancient Kalinga—modern Puri District, including
Bhubaneswar, or ancient Tribhuvanesvara, Puri and Konark.
o The main architectural features of Odisha temples are classified in three orders, i.e., rekhapida,
pidhadeul and khakra.
• Rekhapida (Rekha Deula): Rekha in Odia means a straight line. The typical Rekhapida complex has
a shrine with different parts of the temple that align along the same line or axis.
o It is a tall building with a prominent curvilinear roof (shikhara or vimana) resembling the peak of a
mountain, which covers and typically protects the rectilinear sanctum sanctorum (garbagriha – shrine
that houses the deity).
o These temples may have other structures as a part of a larger temple plan or house a singular deity as a
complete small temple.
o These are typically dedicated to Vishnu, Surya and Shiva.
o Most of the Kalinga temples fall under the Rekha category.
o The most distinct examples of Rekhapida-type temples are the Jagannath Temple at Puri or the Lingaraj
Temple at Bhubaneswar.
• Pidhadeul (Pidha Deula): Pidhadeul refers to the square building with a pyramidal-shaped roof,
which consists of Pidhas (horizontal platforms) that are layered upon each other to form the roof,
arranged into three tiers.
o Typically, these structures are seen as supporting structures for a Rekhapida style of temple with
a Jagamohan (assembly or congregation hall), and often a Natyamandir (dance hall) and
a Bhogamandapa (hall of offerings) are built in this style.
o Pidha-style is usually used for jagamohanas and is only very rarely used to house the deity.
o The assembly hall of the Konark Sun Temple is an outstanding example of Pidhadeul-type temples.
• Khakra (Khakhara Deula): The Khakra is a different type of structure quite similar to the Dravidian
Gopuram design.
o It is a rectangular building with a truncated pyramid-shaped roof, like the gopuras.
o Very few temples depict Khakra style; usually, these temples are dedicated to a form of Devi.
o The Khakra order is represented by the Varahi Temple in Chaurasi and the Vaital Deula in Bhubaneswar
• Hence option (d) is the correct answer.

Q 44.A
• Recent Context: The National Health Authority (NHA) under its flagship scheme Ayushman Bharat
Digital Mission (ABDM) announced successful integration of additional 13 digital health solutions in the
ABDM Sandbox environment. This takes the tally of ABDM integrated service applications to 40 since the
16 www.visionias.in ©Vision IAS
https://www.pdfnotes.co/

.
national launch was announced on Sep 27, 2021. The ABDM partners ecosystem now consists of 16
government applications and 24 private sector applications.
• RCH ANMOL is one of the 13 digital health solutions in ABDM.
o It is an application for tracking pregnant women, infants and children for their health,
vaccination, and nutrition status.
• Hence, option (a) is the correct answer.

Q 45.B
• The Vaikom Satyagraha was launched in 1924. The heart of this protest was at the Vaikom Mahadeva
Temple, where the untouchables were not permitted to use a public road around it.
• In February 1924, the Anti-Untouchability Committee held a public meeting in Vaikom, where it was
decided that the rule barring temple roads to low castes would be defied by a group of satyagrahis. On
March 30 volunteers from different parts of Travancore arrived in Vaikom. Hence, statement 1 is correct.
• Periyar E.V. Ramaswamy was a great social reformer. In 1924, he took an active part in the Vaikam
Satyagraha and provided leadership to the satyagraha.
• In 1925, he started the “Self-Respect Movement”. He denounced the caste system, and child marriage and
enforced widowhood. He himself conducted many marriages without any rituals. Such a marriage was
known as a “Self- Respect Marriage.” Hence, Statement 2 is correct.
• The Non-co-operation Movement was launched in 1920 and was withdrawn by Mahatma Gandhi in
February 1922 after the Chauri Chaura incident. The Vaikom Satyagraha began in February 1924 two
years after the Non-co-operation Movement was withdrawn. However, the Vaikom Satyagraha was
supported by Mahatma Gandhi and it was during the satyagraha that Mahatma Gandhi met Sree Narayana
Guru for the first time. Hence, statement 3 is not correct.

Q 46.C
• The Monetary Policy Committee (MPC) of the Reserve Bank of India on Wednesday hiked the key policy
rate, the Repo rate or the rate at which the RBI lends funds to banks, by 25 basis points to 6.50 percent in a
bid to rein in retail inflation.
• Impact of Increase in repo rate by RBI:
o Lending rates of banks are expected to go up as the cost of funds is expected to rise further. This will
lead to an increase in the borrowing cost in the economy. EMIs on vehicles, homes, and personal
loans will also rise. The external benchmark linked lending rate (EBLR) of banks will rise by 25 bps
— one basis point is one-hundredth of a percentage point— as such loans are linked to the Repo rate.
As much as 43.6 percent of the total loans are now linked to the Repo rate. Hence option 3 is not
correct.
o The rate of deposit would also get increased by some margin, making the FDs more attractive and
providing a source of fixed income. Hence, increased rates are beneficial for such consumers who
have savings and fixed deposits. Hence option 1 is correct.
o Borrowing money from a bank becomes more expensive as a result, slowing investment and money
supply in the market. The real estate sector, which has seen a good pickup in sales due to low financing
costs, may be adversely affected by the RBI's rate hike step. The increase in RBI's rate will also affect
residential sales growth. In response to the ongoing rise in raw material costs, builders throughout the
country have already raised real estate prices. This may negatively impact the demand in real estate
sector. Hence option 2 is not correct.

Q 47.B
• The brief period between sunrise and full daylight is called dawn and that between sunset and complete
darkness is termed twilight. This is caused by the fact that during periods of dawn and twilight, the earth
receives diffused or refracted light from the Sun while it is still below the horizon. Since the sun rises and
sets in the vertical path at the equator, the period during which refracted light is received a short.
But in temperate latitudes, the sun rises and set in an oblique path and the period of refracted light
is longer. It is much longer still at the poles so the winter darkness is really only twilight most of the
time.
• Earth is tilted at 66.5 degrees with respect to the orbital plane. This changes the apparent altitude of the Mid
Day sun. The sun is vertically overhead at the equator on two days each year. These are usually the
21st of March and 21st of September though the date changes because the year is not exactly 365
days. These two days are termed equinoxes. After the March Equinox, the sun appears to move north
and is vertically overhead at the Tropic of Cancer on about the 21st of June. This is known as the summer
solstice. By 22 December the sun will be overhead at the Tropic of Capricorn. this is called Winter solstice.
17 www.visionias.in ©Vision IAS
https://www.pdfnotes.co/

.
The tropics do Mark the limits of the overhead sun, for beyond these, the sun is never overhead at any time
of the year. Hence, option (b) is the correct answer.
• The latitudes and longitudes are commonly referred to as geographical coordinates as they provide a
systematic network of lines upon which the position of various surface features of the earth, can be
represented. With the help of these coordinates, the location, distance, and direction of various points can
be easily determined. Lines of longitude, also called meridians, are imaginary lines that divide the
Earth. They run north to south from pole to pole, but they measure the distance east or west.
Longitude is measured in degrees, minutes, and seconds. Although latitude lines are always equally
spaced, longitude lines are furthest from each other at the equator and meet at the poles.
• As Earth rotates, it wobbles slightly upon its axis, like a slightly off-center spinning toy top. This wobble is
due to tidal forces caused by the gravitational influences of the Sun and Moon that cause Earth to bulge at
the equator, affecting its rotation. The trend in the direction of this wobble relative to the fixed positions of
stars is known as axial precession. The cycle of axial precession spans about 25,771.5 years. Axial
precession makes seasonal contrasts more extreme in one hemisphere and less extreme in the
other. Currently, perihelion occurs during winter in the Northern Hemisphere and in summer in the
Southern Hemisphere. This makes Southern Hemisphere summers hotter and moderates Northern
Hemisphere seasonal variations. But in about 13,000 years, axial precession will cause these conditions to
flip, with the Northern Hemisphere seeing more extremes in solar radiation and the Southern Hemisphere
experiencing more moderate seasonal variations.

Q 48.B
• The Department of Fisheries, Ministry of Fisheries, Animal Husbandry and Dairying, Government
of India has launched the ‘Sagar Parikrama’ program on occasion of 75th Azadi Ka Amrit
Mahotsav. Sagar Parikrama program is being organized through a pre-decided sea route covering coastal
states/UTs. Hence, statement 1 is not correct.
• The fishermen, fisher communities and stakeholders in the coastal areas are likely to be benefitted with
awareness of various beneficiary oriented schemes and programs being implemented by the Government
and showcasing them the best practices that may be adopted.
• The interaction programs being organized during the ‘Sagar Parikrama’ aims to resolve the issues of the
fishers and other stakeholders and to facilitate their economic upliftment through various fisheries schemes
and programs being implemented by the Government of India.
• The main objectives of ‘Sagar Parikrama’ are
o to facilitate interaction with fishermen, coastal communities and stakeholders so as to disseminate
information of various fisheries related schemes and programs being implemented by the
Government; Hence, statement 2 is correct.
o demonstrating solidarity with all fisher folk, fish farmers and concerned stakeholder as a spirit of
Aatmanirbhar Bharat;
o to promote responsible fisheries with focus on sustainable balance between the utilization of marine
fisheries resources for food security of nation and livelihoods of coastal fisher communities and
o protection of marine ecosystems.

Q 49.C
• The Great Blue Wall Initiative is a regionally important initiative that seeks to create a network of
marine conservation across the Western Indian Ocean.
• What is the project?
o It is an international movement that seeks to protect and restore the Indian Ocean. Ten countries
are coming together to commit towards a larger global pledge to protect 30 percent of the world's
oceans from the effects of Climate change and Global warming. These countries are planning
to create a connected network of marine conversation areas that protect some of the most
threatened aquatic areas in the world. Hence statement 1 is correct.
o The initiative took inspiration from the Great Green Wall Project which seeks to reforest a length
of 8,000 km across Africa and to prevent further desertification and deterioration of the sub-Saharan
region.
o The initiative will protect not just ecologically-important marine areas like coral reefs but also
environmentally-critical areas like marshlands, mangroves, seagrass meadows, and underwater
ecosystems
• The initiative was formally launched during the COP26 summit in Glasgow by Seychelles, though it
was announced a month prior during the IUCN World Congress. Hence statement 2 is correct.

18 www.visionias.in ©Vision IAS


https://www.pdfnotes.co/

.
Q 50.B
• Islington Commission, 1912 a Royal Commission formed to recommend reforms in the Public Service
of British India with Lord Islington as its chairman. A ten member commission, which included three
Indians. Hence pair 1 is correctly matched.
o Islington Commission was given the task of examining the Indian demand to increase the number
of Indians in the Higher Civil Services and, most importantly, holding simultaneous examination
in India.
o The commission was guided by three principal considerations: firstly, to maintain high standards of
civil administration in British India; secondly, to safeguard the paramount interest of British rule; and
thirdly, to satisfy the reasonable aspirations of Indians and promote friendly relations between Indians
and Europeans for better governance. The commission completed its report in 1915, but owing to the
outbreak of the First World War, the report was not published until 1917.
o The recommendations of the report became defunct in 1918, when the montagu-chelmsford report
proposed Indian appointment to one-third of the posts and action was taken in favour of holding
simultaneous ISC examination in London and New Delhi in 1922.
• Muddiman Committee, 1924 a committee appointed by the Government of India in early 1924 with
the terms of reference of making an empirical investigation into the working of the Constitution as
set up in 1921 under the India Act of 1919, and making appropriate recommendations for the
consideration of the authorities. The immediate background to the committee was the mounting political
unrest on the dyarchy issue of the Constitution. The official designation of the committee was the Reforms
Enquiry Committee, but it came to be known as the Muddiman Committee after the name of its chairman,
Sir Alexander Muddiman, who was then a Home Member of the Government of India. Hence pair 2 is not
correctly matched.
• Lee Commission, was appointed by the British government in 1923 to consider the ethnic composition
of the superior Indian public services of the government of India. The chairman was Lord Lee of
Fareham, and there were equal numbers of Indian and British members. The Lee Commission proposed in
1924 that 40 percent of future entrants should be British, 40 percent Indians directly recruited, and 20
percent Indians promoted from the provincial service. By the date of independence in 1947, more than half
the service of about 1,000 members were Indians, many with long experience and holding high positions.
o In 1902, Raleigh Commission was set up to go into conditions and prospects of universities in India
and to suggest measures for improvement in their constitution and working. The commission precluded
from reporting on primary or secondary education. It is also known as the Indian Universities
Commission of 1902, its major recommendations were the reform of school education, curricular
reforms at universities, recommendations on education and examinations and state scholarships. Based
on its recommendations, the Indian Universities Act was passed in 1904. Hence, pair 3 is not correctly
matched.

Q 51.D
• A nongovernmental organization (NGO), is a voluntary group of individuals or organizations, usually not
affiliated with any government, that is formed to provide services or to advocate a public policy. Although
some NGOs are for-profit corporations, the vast majority are non-profit organizations.
• Constitutional and legal provisions related to NGOs:
o 7th Schedule - The concurrent List mentions charitable institutions, charitable and religious
institutions. Hence option 1 is correct.
o Article 19(1)(c) - Allows the right to form associations. Hence option 2 is correct.
o The Foreign Contribution Regulation Act (FCRA) - This act regulates the inflow of foreign
contributions to NGOs in India. Hence, option 3 is correct.
o The Societies Registration Act, 1860 - This act provides for the registration of non-profit organizations
in India. Hence option 4 is correct.
o The Companies Act, 2013 - This act governs the formation and operation of companies in India,
including Section 8 companies, which are non-profit organizations. Hence option 5 is correct.

Q 52.C
• Although Harappan sites share certain common features, there are also significant regional and inter-
site differences. These are visible, for instance, in the frequency of various funerary practices across sites.
o The post-cremation burials were much more numerous at Harappa than at Mohenjodaro.
o Symbolic burials with grave goods but no skeletons were found at Kalibangan.
o Fractional burials (where the body was exposed to the elements and the bones then gathered and buried)
were found at Mohenjodaro and Harappa.
19 www.visionias.in ©Vision IAS
https://www.pdfnotes.co/

.
o A triangular terracotta cake found at Kalibangan has a carving of a horned deity on one side and an
animal being dragged by a rope by a human on the other. The latter has been interpreted as suggesting
the practice of animal sacrifice. Further, the most striking evidence suggesting ritualistic practices
comes from the ‘fire altars’ found on the citadel mound at Kalibangan.
o All this suggests a variety of subsistence strategies, food habits, craft traditions, religious beliefs, cultic
practices, and social customs. Hence statement 1 is not correct.
• The Harappan cities did not possess the necessary raw materials for the commodities they produced. They
did not use metallic money. We have no idea about their currency. Most probably, they carried out all
exchanges through barter. In exchange for finished goods, they transported metals from neighbouring areas
using boats and bullock carts. They practised navigation on the coast of the Arabian Sea. They had
commercial links with Rajasthan, Afghanistan and Iran. Their cities also carried on commerce with
those in the land of the Tigris and Euphrates. Many Harappan seals have been discovered in
Mesopotamia. Hence statement 2 is correct.
• The Harappan villages, mostly situated near the floodplains, produced sufficient foodgrains not only to feed
themselves but also the town people. The Indus people produced wheat, barley, peas, etc. Although the
Harappans practised agriculture, animals were kept on a large scale. Oxen, buffaloes, goats, sheep, pigs ,
dogs and cats were domesticated. Hence statement 3 is not correct.

Q 53.D
• Kahramanmaras is a city in southern Turkey. It is situated at the edge of a fertile plain below Ahır
Mountain, east-northeast of Adana. The city is near the southern outlet of three important passes through
the Taurus Mountains. Recently, a 7.7 magnitude earthquake (EQ) with the epicenter of Pazarcık,
Kahramanmaraş happened in Türkiye's South-East Anatolia Region. Hence pair 1 is correctly matched.
• Damascus is the capital of Syria. Damascus is one of the oldest continuously inhabited cities in the world.
An Israeli missile strike killed and destroyed a residential building in the Syrian capital of Damascus. The
strike hit in Kafr Sousa, a high-security area of the Syrian capital. Hence pair 2 is correctly matched.
• A massive underwater volcanic eruption was recorded in the small island country of Tonga on January 15,
2022. Japan's Meteorological Agency had issued tsunami warnings for the Amami Islands and the Tokara
island chain in Kagoshima Prefecture. The Tokara Islands are a small archipelago in Japan, south of
Kyushu and north of the Amami Islands. They are also sometimes known as the Toshima Islands. Hence
pair 3 is correctly matched.

Q 54.A
• Rotterdam Convention on Prior Informed Consent Procedures (PIC) entered into force in 2004.It is
an international treaty designed to facilitate informed decision-making by countries with regard to trade in
hazardous chemical. It is a legally binding instrument. India acceded to the Convention in 2006.
• It focuses on an open exchange of information and calls on exporters of hazardous chemicals to use proper
labelling, include directions on safe handling, and inform purchasers of any known restrictions or bans.
• Annex III lists chemicals that have been banned or severely restricted for health or environmental
reasons by two or more Parties. Hence statement 1 is correct.
• The Conference of the Parties has decided to subject these chemicals to the PIC procedure. The convention
has 161 parties, which includes 158 UN member states, the Cook Islands, the State of Palestine, and the
European Union. Non-member states include the United States. India ratified the Convention in 2005.
• International trade of two new hazardous pesticides — Iprodione and Terbufos — has been
recommended for “prior informed consent” (PIC) procedure under Rotterdam convention. Hence
statement 2 is not correct.
• The recommendations for PIC were made at 18th meeting of the Rotterdam Convention’s Chemical Review
Committee (CRC 18) held at Rome, Italy.
• About the pesticides:
o Iprodione is a fungicide used on vines, fruits, trees and vegetables.
o It has been classified as carcinogenic and toxic for reproduction. Terbufos is a soil insecticide used
commonly on sorghum, maize, beet and potatoes.
o It has also been found to pose risk to aquatic organisms due to its toxicity.
o In India, the use of these chemicals was permitted by the 2015 Anupam Verma committee report.
o India is one the largest exporters of Terbufos.

20 www.visionias.in ©Vision IAS


https://www.pdfnotes.co/

.
Q 55.B
• Scientists have observed the thermal Hall effect in insulators, leading to an important open question in
condensed-matter physics.
o Thermal Hall effect emerges in an electrical conductor in the following way. when one applies a
temperature gradient in an electrical conductor in one direction, another temperature gradient appears
in a perpendicular direction in the presence of a magnetic field. The electrons in the material carry both
electric charges and thermal energy, and the magnetic field deflects them, giving rise to the
perpendicular gradient.
• But scientists have observed the thermal Hall effect in insulators as well, especially terbium oxides,
strontium titanate and two kinds of cuprates. Electrons in insulators aren’t involved in transferring
heat or electricity, so what could explain this? This is an important open question in condensed-matter
physics. A leading candidate for an answer involves a particle called the phonon.
• Technically, phonons aren’t particles; they’re quasiparticles – packets of energy that behave like
particles in a system. A phonon is a quasiparticle of vibrational energy. When the grid of atoms that
make up the material vibrates, it releases this energy, and physicists encapsulate it in the form of
phonons. Hence statement 1 is not correct.
• In the presence of a magnetic field, electrons are deflected from their paths in a perpendicular direction.
While phonons can’t be deflected by the magnetic field – they have no electric charge – they are
affected by the electrons that are deflected by the magnetic field. When they scatter off the deflected
electrons, they are deflected in a perpendicular direction. Hence statement 2 is correct.
• The thermal Hall resistivity of the phonons provides quite unobscured access to the magnitude of these
couplings, which could provide quantitative insights into the mechanisms leading to superconductivity.

Q 56.D
• Karpuramanjari is a drama written by Rajashekhara in Prakrit. Rajashekhara has been highly
esteemed for his proficiency in the Prakrit. He was the court poet of the Gurjara Pratiharas.
Rajashekhara also wrote the Kavyamimamsa. The work is essentially a practical guide for poets that
explains the elements and composition of a good poem.
o Karpuramanjari contains four Acts called Javanikantara. It tells us how king Candapala marries
Karpura-Manjari,the daughter of the Kuntala King, and thus becomes a paramount sovereign.
o The jealousy of the queen, and the machinations that bring the king and the heroine together, from the
plot of the play, the Adbhuta Rasa is represented by the sorcerer Bhairvananda and his tricks.
• Kathasaritsagara was composed around 1070 CE by a Kashmiri Shaivite Brahmin called
Somadeva. In a short poem at the end of his work, Somadeva states that he was the court poet of King
Anantadeva of Kashmir and that he composed his Kathasaritsagara for the amusement of Queen
Suryavati, wife of King Anantadeva, to distract her mind from its usual occupation of ‘worshipping Shiva
and acquiring learning from the great books.
• Brihatkatha is a mammoth work composed by Gunadhya. It is a work that has been given the highest
rank beside epics like Mahabharata and Ramayana. It is a vast collection of tales dating from the
early centuries of the common era. The stories generally had a secular character and imparted
nuggets of wisdom. Nothing is definitively known about Gunadhya's life
o He composed Brihatkatha based on three sources. From Ramayana, he got the motif of the search of a
husband for a wife cruelly stolen from him soon after a happy marriage. From Buddhist legends and
other traditions of Ujjain and Kausambi, he was familiar with the tales of Pradyuta, Mahasena and
Udayana. He also knew many tales of sea voyages and strange adventures. From the latter source and
Buddhist legends, he derived the conception of the monarch who is the secular counterpart of the
Buddha.
• Hence all pairs are correctly matched and option (d) is the correct answer.

Q 57.A
• India is the 7th largest country in the world by area. India accounts for 2.4 percent of the total world surface
area. The rea of India is 3,287,469 square km. Rajasthan (342,239 sq km) is the largest state in India in
terms of area. Rajasthan accounts for 10.4 percent of the total country's area. Rajasthan is followed
by Madhya Pradesh and Maharashtra having areas of more than 3 lakh sq km.
• Hemis National Park is located at a high altitude in eastern Ladakh. The park was established in 1981
and the total area of the park is 3,350 sq km. This park is the biggest in India and is considered one of the
biggest national parks in South Asia. The park is home to several species of endangered mammals and birds.
Hemis National Park is a protected area for endangered mammals such as snow leopards, leopards, Asiatic
ibex, Tibetan wolf, the Eurasian brown bear, and the red fox. Hence, statement 1 is not correct.
21 www.visionias.in ©Vision IAS
https://www.pdfnotes.co/

.
• The total length of India’s coastline is 7516.6 kilometers. Of this, the mainland coastline length is 5422.6
kilometers whereas the island territories' coastline length is 2094 kilometers. States/UTs having coastlines
are Gujarat, Maharashtra, Goa, Daman and Diu, Karnataka, Kerala, Tamil Nadu, Puducherry, Andhra
Pradesh, Odisha, and West Bengal. Island territories having coastlines are – Andaman & Nicobar Islands
and Lakshadweep Islands. Gujarat has the longest mainland coastline in India.
• The Asiatic lion is one of the most amazing, and powerful big cats found in India. The Panthera Leo’s
population of lions in India is found in Gujarat. More precisely, Asiatic Lions are now confined to
Sasan Gir National Park in Gujarat, India. Hence, statement 2 is correct.
• According to Census 2011, Arunachal Pradesh is the state with the lowest population density in India. It
has a total population density of 17 Within an area of 83743 sq. km.
• There are 75 groups of most vulnerable sections amongst the tribals who are classified as the
Particularly Vulnerable Tribal Groups (PVTGs) spread over 18 States and the Union Territory of
Andaman & Nicobar Islands. Among the 75 listed PVTGs, the highest number are found in Odisha
(13), followed by Andhra Pradesh (12), Bihar including Jharkhand (9) Madhya Pradesh including
Chhattisgarh (7) Tamil Nadu (6) Kerala and Gujarat having five groups each. The remaining PVTGs
live in West Bengal (3) Maharashtra (3), two each in Karnataka and Uttarakhand and one each in Rajasthan,
Tripura and Manipur. Hence, statement 3 is not correct.

Q 58.B
• The Asian Development Bank (ADB) is a regional development bank established on 19 December 1966,
which is headquartered in the Ortigas Center located in the city of Mandaluyong, Metro Manila, Philippines.
The bank also maintains 31 field offices around the world to promote social and economic development in
Asia. Hence statement 1 is not correct.
• As of 31 December 2021, ADB's five largest shareholders are Japan and the United States (each with 15.6%
of total shares), the People's Republic of China (6.4%), India (6.3%), and Australia (5.8%). Hence
statement 2 is not correct.
• From 31 members at its establishment, ADB now has 68 members. Its members are from different
continents; e.g. North America, Western Europe. The ADB was modeled closely on the World Bank, and
has a similar weighted voting system where votes are distributed in proportion with members' capital
subscriptions. Hence statement 3 is correct.
• The highest policy-making body of the bank is the Board of Governors, composed of one representative
from each member state. The Board of Governors, in turn, elect among themselves the twelve members of
the board of directors and their deputies. Eight of the twelve members come from regional (Asia-Pacific)
members while the others come from non-regional members.

Q 59.C
• While banks manage the majority of ATMs in India, there is a relatively new category of ATMs that are
not branded or managed by banks. These include White label ATMs that are set up, owned, and operated
by non-banks.
• White Label ATMs are ATMs that are owned and operated by non-banking entities. ATMs operated under
this business model allow customers to use them for banking transactions regardless of the bank they have
an account. Hence statement 1 is correct.
• The Reserve Bank of India (RBI) authorizes non-bank ATM operators under the Payment and Settlement
Systems Act, 2007. Hence statement 3 is not correct.
• Need for WLAs: Banks generally have to incur various costs to set up ATMs. It may include ATM systems,
security, and cash management costs. There are many places where a bank branch may not exist, and in
such a case, the bank might not find it lucrative to have its own ATM. So, RBI allowed WLAs to operate.
• Services offered: Besides dispensing cash, WLAs may also offer many other services and facilities to users.
Such services include account information, cash deposit, bill payment, mini statement, PIN change, and
placing requests for chequebooks.
• Every time, a customer of a particular bank uses an ATM deployed by another bank, the former bank will
have to pay a fee to the other bank. This is called an interchange fee. For years, private banks and white-
label ATM operators had been seeking an increase in the interchange fee.
• Recently, WLA operators have urged the Reserve Bank of India to link the interchange fee to either
a price index or any other suitable index. So, if the costs go down, the interchange fee goes down, and
vice versa. Hence statement 2 is not correct.

22 www.visionias.in ©Vision IAS


https://www.pdfnotes.co/

.
Q 60.B
• Puranas as a sourcebook of Indian painting: The third Khanda of the Vishnudharmottara Purana, a
fifth-century text has a chapter Chitrasutra, which should be considered a sourcebook of Indian art in
general and painting specifically. It talks about the art of image-making called Pratima lakshana, which
are canons of painting. The Khanda also deals with the techniques, tools, material(s), surface (wall),
perception, perspective, and three-dimensionality of human figures. Hence option (b) is the correct
answer.
• Different limbs of painting, such as roopbheda or looks and appearance; pramana or measurements,
proportion, and structure; bhava or expressions; lavanya yojana or aesthetic composition; sadrishya or
resemblance; and varnikabhanga or use of brush and colors have been explained at length with examples.
Each of these has many sub-sections. These canons were read and understood by artists and followed
through centuries, thus, becoming the basis of all styles and schools of painting in India.

Q 61.C
• The Indian National Congress was founded at its first session held in Bombay in 1885. W.C. Banerjee
was its first president. The early Congress leadership known as Moderates had total faith in the British
sense of justice and fair play. The Moderates used petitions, resolutions, meetings, leaflets and pamphlets,
memorandum and delegations to present their demands.
• In the beginning, the British Government welcomed the birth of the Indian National Congress. In 1886,
Governor General Lord Dufferin gave a tea garden party for the Congress members in Calcutta. The
government officials had also attended Congress sessions. With the increase in Congress demands, the
government became unfriendly and consequently during the later years government officials were
prohibited from attending the sessions of the INC. Hence, statement 1 is not correct.
• The Moderate leadership did believe in social reforms and many moderate leaders actively sought to
reform society. For instance, some Moderates like Ranade and Gokhale protested against child
marriage and widowhood. However, the moderates did not advocate Social reforms officially from the
forum of the Indian National Congress. The reforms were left to different classes and caste
organizations.
• M.G. Ranade and Raghunath Rao founded Indian Social Conference for social reforms. It met
annually from its first session in Madras in 1887 at the same time and venue as the Indian National Congress.
It focused attention on the social issues of importance; it could be called the social reform cell of the Indian
National Congress. Hence, statement 2 is not correct.
• The major demands of the Congress during its early phase were: Expansion and reform of legislative
councils, Greater opportunities for Indians in higher posts by holding the ICS examination simultaneously
in England and in India, Separation of the judiciary from the executive, Reduction of land revenue and
protection of peasants from unjust landlords, Abolition of salt tax and sugar duty, Reduction of spending
on army, Freedom of speech and expression and freedom to form associations. Hence, statement 3 is
correct.

Q 62.A
• Quantum dots (QDs) are nanoscale crystals that exhibit unique optical and electronic properties.
These artificially synthesized semiconductor nanoparticles have a wide range of potential applications,
including use in composites, solar cells, fluorescent biological labeling, displays, lighting, and medical
imaging. They can absorb ultraviolet (UV) light and produce sharp, bright colours, making them especially
attractive for TVs, smartphones and LEDs. Hence statement 1 is correct.
• They are extremely tiny particles with properties vastly superior to conventional semiconductors. They
are capable of emitting monochromatic light naturally when they are excited by an energy source.
When activated by UV light, they can produce visible light in different colours depending on their
size. Small dots produce blue light, for example, while large ones radiate red. Hence statement 2 is
correct.
• Quantum dots are normally made in industrial settings with high temperatures and toxic, expensive
solvents—a process that is neither economical nor environmentally friendly. But recently, researchers have
now pulled off the process using water as a solvent, making a stable end-product at room temperature.
They discovered the first known de novo (newly created) protein that catalyzes the synthesis of
quantum dots. Their work opens the door to making nanomaterials in a more sustainable way by
demonstrating that protein sequences not derived from nature can be used to synthesize functional
materials—with pronounced benefits to the environment. Hence statement 3 is not correct.

23 www.visionias.in ©Vision IAS


https://www.pdfnotes.co/

.
Q 63.C
• Alternative Dispute Resolution ("ADR") refers to any means of settling disputes outside of the courtroom.
ADR typically includes early neutral evaluation, negotiation, conciliation, mediation, and arbitration. As
burgeoning court queues, rising costs of litigation, and time delays continue to plague litigants, more states
have begun experimenting with ADR programs. Some of these programs are voluntary; others are
mandatory.
• Negotiation
o Negotiation is the preeminent mode of dispute resolution. While the two most known forms of ADR
are arbitration and mediation, negotiation is almost always attempted first to resolve a dispute.
Negotiation allows the parties to meet in order to settle a dispute. The main advantage of this form of
dispute settlement is that it allows the parties themselves to control the process and the solution.
Negotiation is much less formal than other types of ADRs and allows for a lot of flexibility.
• Mediation
o Mediation is also an informal alternative to litigation. Mediators are individuals trained in
negotiations, who bring opposing parties together and attempt to work out a settlement or
agreement that both parties accept or reject. Mediation is not binding. Mediation is used for a wide
gamut of case types ranging from juvenile felonies to federal government negotiations with Native
American Indian tribes. Mediation has also become a significant method for resolving disputes between
investors and their stock brokers. See Securities Dispute Resolution. Hence, statement 2 is correct.
• Arbitration
o Arbitration is one of the most emblematic and growing forms of ADR. Arbitration is more formal than
mediation and has a lot of similarities with traditional court proceedings, involving limited discovery
and simplified rules of evidence (ex. hearsay is usually admissible in arbitration). Hence, statement 1
is not correct.
• Conciliation means the attempted resolution of issues raised by a complaint, or by the investigation of a
complaint, through informal negotiations involving the aggrieved person, the respondent, and the Assistant
Secretary. Hence, statement 3 is not correct.

Q 64.A
• The U.S. dollar is the currency most used to purchase oil. Because oil has such a high level of demand,
some oil-exporting countries could build up so many dollars they cannot spend them all. These dollars were
historically called petrodollars. Simply, petrodollars are U.S. dollars paid to an oil-exporting country.
• Some countries exported so much oil and accumulated such large amounts of U.S. dollars that they couldn't
spend them all. If the country accepting U.S. dollars as payment can't spend them all, they are effectually
removed from circulation. The dollars these countries could not spend came to be known as petrodollars.
The United States uses the power of petrodollars to enforce its foreign policy.
• A serious challenge to the petrodollar comes from moves to trade oil and gas outside the dollar zone. This
move can cut into the monopoly of the dollar that the U.S. has since the 1970s. Many oil producers, refiners
and buyers such as Russia, India, China, Venezuela and Iran have initiated trading hydrocarbons in national
currencies. India’s vostro accounts enable rupee payments for Russian crude. China is using the yuan-ruble
route for Russian oil since 2020. China is in talks with Saudi Arabia and the Gulf countries to trade oil in
yuan.
• Hence option (a) is the correct answer.

Q 65.C
• Mauna Loa, the world's largest active volcano, erupted in Hawaii marking its first eruption in nearly
40 years. The last eruption was in 1984. Hence, statements 1 and 3 are correct.
• Mauna Loa is what’s known as a shield volcano, formed by repeated fluid lava flows that tend to give
the volcano a gently sloping, rounded profile like that of a shield. That is where Mauna Loa gets its
name — it means “long mountain” in Hawaiian. That is in contrast to composite volcanoes like Mount
Hood in Oregon, Mount Rainier in Washington, or Mount Fuji in Japan which have what to many is a more
classic volcano shape, with tall, steep sides created by repeating layers of lava flows, volcanic ash and
cinders. Hence, statement 2 is not correct.
• Any volcano that has erupted within the Holocene period (in the last 11,650 years) is considered to be
“active” by scientists. “Dormant” volcanoes are those active volcanoes which are not in the process of
erupting currently but have the potential to do so in the future. Mauna Loa was a dormant volcano for the
last 38 years. “Extinct” volcanoes are ones which scientists predict will never face any further volcanic
activity. Ben Nevis, the tallest mountain in the UK, is an extinct volcano.

24 www.visionias.in ©Vision IAS


https://www.pdfnotes.co/

.
• Other important volcanoes are:
o Mount Vesuvius, Italy: In 79 CE, Mount Vesuvius erupted, in one of the deadliest eruptions in
European history, killing as many as 16,000 and destroying the town of Pompeii. According to
scientists, the explosion released 100,000 times the thermal energy that was released with the atomic
bombings at Hiroshima and Nagasaki. It is said to have instantly boiled the blood of all those who were
too close to it.
o Mount Fuji, Japan: A defining image of Japan, Mount Fuji towers over the countryside with its snow-
capped peaks and barren surface. It last erupted in 1707-1708 and had a devastating effect on the local
population. The tephra release led to a significant agricultural decline, leading to widespread starvation
in the Edo (now Tokyo) area. Although this eruption itself did not directly kill a lot of people, its
subsequent impact proved deadly.
o Eyjafjallajökull, Iceland: Sometimes referred to as E15, it is one of the many volcanic features of
Iceland. In 2010, a relatively small eruption managed to bring air traffic in Europe to a complete
standstill. 20 countries closed their airspace, impacting approximately 10 million travellers.
o Kīlauea, Hawaii: Adjacent to the Mauna Loa, this is one of the most active volcanoes on the planet. It
has been erupting intermittently since recorded history, with its eruption lasting from 1983 to 2018
being the longest continuous eruption ever recorded. It is a major tourist attraction, with the earliest
hotel built at the edge of the volcano in the 1840s.
o Mount St Helens, USA: Located in Washington State, Mount St. Helens was a major eruption that
occurred on May 18, 1980, and it remains the deadliest and most economically destructive volcanic
event in U.S. history. It started after an earthquake hit the region, killing 57 and causing property
damage of over $1 billion. It remains an active volcano and one that is considered to be amongst the
riskiest by scientists.

Q 66.C
• There are a large number of turtles in India. This comes as no surprise as India is the seventh-largest country
in the world in terms of land size.
o In all, there are about 25 turtle species in India with several of the species on the verge of extinction.
Some of the species facing extinction include the Asian Giant softshell, Leith’s softshell, red-crowned
roofed turtle, northern river terrapin, and black softshell.
• Red-Crowned Roofed Turtle:
o The red-crowned roofed turtle is a large freshwater turtle that can reach a length of 20 inches.
o During the breeding season, males have red stripes on the neck. All other times, the males are brown
or olive. Females are always brown or olive in coloration.
o The species can be found in the Chambal National Park which is located in Madhya Pradesh. The
species inhabits the Ganges River.
o The red-crowned roofed turtle is a critically endangered turtle on the IUCN Red List. Recently in COP
19 CITES, India’s proposal for the transfer of Red-Crowned Roofed Turtles from Appendix II
to Appendix I of CITES has been adopted. Hence pair 2 is not correctly matched.
• Indian Softshell Turtle:
o The Indian softshell is a giant turtle that can reach lengths of 28 inches.
o Although this turtle isn’t as huge as the Asian giant softshell, it is nevertheless a huge turtle. This turtle
can be found in large reservoirs and large river systems such as the Mahanadi, Indus, and Ganges
Rivers. It is also known as Nilssonia Gangetica or Gange's softshell turtle..
o The species occurs in Chambal National Park in Madhya Pradesh, Dudhwa Tiger Reserve in Uttar
Pradesh, Harike Lake in Punjab, and Keoladeo National Park in Rajasthan.
o The turtle is listed as Endangered in the IUCN Red List of Threatened Species. The species is also
listed in CITES I which means that international trade of the species is prohibited. Regardless of the
turtle’s conservation status, it is still exploited within its geographic range. Hence pair 1 is not
correctly matched.
• Indian Black Turtle.
o The Indian black turtle is a semi-aquatic turtle that can be found in ponds, rivers, and forests.
o The turtle is endemic to most of the protected areas within its geographic range. These
include Nongkhyllem Wildlife Sanctuary in Meghalaya, Corbett Tiger Reserve & Dudhwa Tiger
Reserve in Uttar Pradesh, Manas Tiger Reserve in Assam, and Point Calimere Wildlife Sanctuary
& Mudumalai Wildlife Sanctuary in Tamil Nadu.
o It is listed on the IUCN Red List as of Least Concern. It was also listed in the CITES Appendix II in
2013. It is not included in the Schedules of the WLPA (Indian Wildlife Protection Act). Hence pair
3 is correctly matched.
25 www.visionias.in ©Vision IAS
https://www.pdfnotes.co/

.
Q 67.D
• A recent study showed that Kelp forests — underwater ecosystems formed in shallow water by the dense
growth of several different species known as kelps — are declining because of climate change. Kelp
populations at equatorward-range edges are particularly vulnerable to climate change as these locations are
undergoing warming at or beyond thermal tolerance thresholds, the study published in the journal Nature.
Due to this, the unique adaptive or evolutionary genetic diversity that the rear-edge populations (populations
in warm, low-latitudes) may contain is also under threat due to rapid warming.
• Kelp Forests are underwater ecosystems formed in shallow water by the dense growth of several different
species known as kelps. Though they look very much like plants, kelps are actually extremely large
brown algae. Some species can reach heights (underwater) of 45 m, and under ideal physical conditions,
kelp can grow 45 cm in a single day. As a result of this incredible growth, kelp forests can develop very
quickly in areas where they did not previously exist. Hence, statement 1 is not correct.
• Kelp thrives in cold, nutrient-rich waters. Because kelp attaches to the seafloor and eventually grows to the
water’s surface and relies on sunlight to generate food and energy, kelp forests are always coastal and
require shallow, relatively clear water. Generally speaking, kelps live further from the tropics than coral
reefs, mangrove forests, and warm-water seagrass beds, so kelp forests do not overlap with those
systems. Like those systems, though, kelp forests provide important three-dimensional, underwater
habitat that is home to hundreds or thousands of species of invertebrates, fishes, and other algae.
Some species aggregate and spawn in kelp forests or utilize these areas as juvenile nursery habitats.
Large predatory species of sharks and marine mammals are known to hunt in the long corridors that form
in kelp forests between rows of individual plants. Hence, statement 2 is not correct.
• Kelp forests comprise one of the ocean’s most diverse ecosystems. Many fish species use kelp forests as
nurseries for their young, while seabirds and marine mammals like sea lions, sea otters and even gray whales
use them as shelter from predators and storms.
• Sea urchins can destroy entire kelp forests at a rate of 30 feet (9 m) per month by moving in herds. Sea
otters play a key role in stabilizing sea urchin populations so that kelp forests may thrive
• Giant kelp is harvested from kelp forests and used as a binding agent in products like ice cream,
cereal, ranch dressing, yogurt, toothpaste, lotion and more. Hence, Statement 3 is correct.

Q 68.B
• NAM stands for the Non-Aligned Movement, and had its origins in the Bandung Conference In
Indonesia. Hence statement 1 is correct.
• NAM is a group of countries that are not formally aligned with any major power bloc or alliance, during
the height of the Cold War, when many countries were forced to choose sides between the United
States and the Soviet Union.
• The Non-Aligned Movement (NAM) does not have a permanent secretariat or headquarters. Instead,
the movement operates through a system of periodic meetings, conferences, and summits held among its
member countries. These events are hosted by member countries on a rotating basis. Hence statement 2 is
not correct.
• The founders of the Non-Aligned Movement (NAM) were Jawaharlal Nehru of India, Kwame
Nkrumah of Ghana, Nasser of Egypt, Sukarno of Indonesia and Tito of Yugoslavia. Hence statement
3 is correct.

Q 69.C
• The World Economic Forum, supported by more than 45 partners today launched the Giving to
Amplify Earth Action (GAEA), a global initiative to fund and grow new and existing public, private,
and philanthropic partnerships (PPPPs) to help unlock the $3 trillion of financing needed each year
to reach net zero, reverse nature loss and restore biodiversity by 2050. Hence, option (c) is the correct
answer.
• With the energy and cost of living crises, the ambition of steering the planet towards a 1.5-degree Celsius
warming pathway hangs in the balance. Meanwhile, the recent agreement at the UN Biodiversity
Conference (CBD COP15) in Montreal to conserve 30% of all earth and sea looks bold but fragile in the
face of a rising biodiversity crisis. Current funding is slow and inadequate, and a new approach is needed
to get capital flowing. Philanthropic giving can address this, with unique qualities not found in other
financings: it is nimble, more tolerant of risks, and is driven by values and long-term outcomes rather than
quarterly returns.
• Philanthropic financing for climate mitigation has risen in recent years, but still represents less than 2
percent of total philanthropic giving, estimated at USD 810 billion in 2021, according to WEF. Greater
philanthropic funding for climate and nature will support, not detract from, existing social priorities.
26 www.visionias.in ©Vision IAS
https://www.pdfnotes.co/

.
Q 70.D
• Recent Context: Memorandum of Understanding (MoU) was signed between Indian Navy's Information
Fusion Centre for Indian Ocean Region (IFC-IOR) and the Regional Coordination Operations Centre
(RCOC), Seychelles.
o The MOU signed aims to promote collaboration towards enhancing maritime domain awareness,
information sharing and expertise development.
• Information Fusion Center - Indian Ocean Region (IFC-IOR), established in 2018, is hosted by Indian
Navy at Gurugram to further Maritime Safety and Security in Indian Ocean Region.
• To enable better correlation and timely inputs, IFC-IOR also hosts International Liaison Officers (ILOs)
from partner nations including Australia, France, Italy, Japan, USA, UK.
• Statement 1 is correct: It was established by the Indian Navy to keep track of shipping traffic. It aims at
strengthening maritime security in the region and beyond, by building a common coherent maritime
situation picture and acting as a maritime security information sharing hub for the region.
• Statement 2 is correct: Its headquarters are presently located at Gurugram, Haryana.
• Statement 3 is correct: It provides regular update regarding monthly weather forecasts and weather
warnings, specific studies/reports are also published by the Centre in this domain.

Q 71.D
• Birsaite Ulgulan (1899-1900)
o In 1899-1900, the Mundas in the region south of Ranchi rose under Birsa Munda. The Ulgulan was
one of the most significant tribal uprisings in the period 1860-1920.
o The rebellion which began as a religious movement gathered political force to fight against introduction
of feudal, zamindari tenures, and exploitation by money-lenders and forest contractors.
o The Mundas claimed Chhotanagpur as their area in 1879. British armed forces were then deployed.
Birsa was captured and imprisoned.
• Maratha Peasant Uprising (1875)
o In 1875, the peasants of Maharashtra mainly from the Poona, Satara and Ahmednagar districts, revolted
against the corrupt practices of Gujarati and Marwari moneylenders.
o The moneylenders used to charge high rates of interest along with manipulating the accounts of poor
peasants.
o In response, the ryots organized the social boycott movement against the moneylenders.
o They refused to buy goods from their shops, work in their homes and fields.
o Soon this social boycott transformed into agrarian unrest.
• Phulaguri Dhawa (1861)
o The farmer uprising in the Phulaguri area of middle Assam in October 1861 AD was one of the early
peasant movements of the Indian freedom movement.
o It was also the first marker of a significant non-cooperation style movement of the Indian freedom
movement wherein the farmers of the Phulaguri region had stopped payment of taxes to the British
administration in open defiance of foreign tyranny.
• Rangpur uprising (1783)
o The peasants themselves often on their own initiative offered resistance to British rule.
o The Rangpur rebellion of 1783 in the northern districts of Bengal is an ideal example of such
opposition.
o In the early days of revenue farming system, the peasantry was oppressed by the revenue contractors
(ijaradars) and company officials, imposing high revenue demands and often collecting illegal cesses
based on Ijaradari system.
o The worst offenders were ijaradars like Debi Singh or Gangagobinda Singh, who had unleashed a reign
of terror in the villages of Rangpur and Dinajpur districts.
• Hence, option (d) is the correct answer.

Q 72.C
• Tropical cyclogenesis requires several favorable precursor environmental conditions. Warm Ocean waters
(of at least 26.5℃ throughout sufficient depth at least on the order of 50 m). Relatively moist layers near
the height of 5 km. Nonnegligible amount of Coriolis force, pre-existing near-surface disturbance. Low
values of vertical wind shear between the surface and upper troposphere. In July and August, winds on
the surface are westerly/south-westerly to the south of the monsoon trough and south-easterly/
easterly to its north and are generally stronger over the seas than the Land areas.
• The upper winds are westerly/south-westerly to the south and south-easterly/ Easterly to the north of this
trough region. Westerly winds increase with height and reach a maximum speed of 20-25 knots between
27 www.visionias.in ©Vision IAS
https://www.pdfnotes.co/

.
900 to 800 hPa levels. Easterly winds strengthen with a height from 200 hPa reaching a maximum of 100
hPa. Speeds are between 60 to 80 knots over the peninsula at 150 /100 hPa level or even at a lower height
(around 200 hPa) in the southern latitude. This results in high values of vertical wind shear which is
unfavorable for Tropical cyclogenesis. So, we don’t get cyclones during main monsoon months like July
and August.
• Hence, option (c) is the correct answer.

Q 73.B
• The 15th Finance Commission has recommended that states be given 41 percent of the divisible tax pool of
the Centre during the period 2021-22 to 2025-26, which is almost at the same level (42 percent) as was
recommended by the 14th Finance Commission. The adjustment of 1% is to provide for the newly formed
union territories of Jammu and Kashmir, and Ladakh from the resources of the center.
• The pool of tax resources of the Union Government to be shared with the states is called the “Divisible
Pool” and it excludes the following items from the gross tax revenue:
o Cost of collection of taxes
o Cess and Surcharge
o Tax Revenue of the Union Territories
o National Calamity Contingent Duty.
• Hence options 1, 2, and 4 are correct.

Q 74.D
• There are five discontinuities inside the earth.
o Conrad Discontinuity: Transition zone between SIAL and SIMA.
o Mohorovicic Discontinuity: Transition zone between the Crust and Mantle.
o Repetti Discontinuity: Transition zone between Outer mantle and Inner mantle.
o Gutenberg Discontinuity: Transition zone between Mantle and Core.
o Lehmann Discontinuity: Transition zone between Outer core and Inner core.

Q 75.D
• Human immune system is a network of tissues, cells, and organs first tries to keep out germs like bacteria,
viruses, fungi, and parasites and then deals with them if they manage to get in. Various components of
immune system are:
• White blood cells are made in bone marrow and are part of the lymphatic system. White blood cells
move through blood and tissue throughout the body, looking for foreign invaders (microbes) such as
bacteria, viruses, parasites and fungi.
• Types of white blood cells are granulocytes (neutrophils, eosinophils, and basophils), monocytes, and
lymphocytes (T cells and B cells).
o When microorganisms, such as bacteria or viruses, enter the body, neutrophils are one of the first
immune cells to respond. They travel to the site of infection, where they destroy the microorganisms
by ingesting them and releasing enzymes that kill them. Neutrophils also boost the response of other
immune cells.
o There are two main types lymphocytes: T cells and B cells. B cells produce antibody molecules that
can latch on and destroy invading viruses or bacteria. T cells are direct fighters of foreign invaders

28 www.visionias.in ©Vision IAS


https://www.pdfnotes.co/

.
and also produced cytokines, which are biological substances that help activate other parts of the
immune system.
o Plasma cell is a type of immune cell that makes large amounts of a specific antibody. Plasma cells
develop from B cells that have been activated. A plasma cell is a type of white blood cell. Hence
option (d) is the correct answer.
• Human leukocyte antigens (HLA) are genes in major histocompatibility complexes (MHC) that help
code for proteins that differentiate between self and non-self. They play a significant role in disease and
immune defense.
• The lymphatic system is made up of:
o lymph nodes (also called lymph glands) – which trap microbes
o lymph vessels – tubes that carry lymph, the colourless fluid that bathes your body's tissues and contains
infection-fighting white blood cells
o white blood cells (lymphocytes)
• The spleen is a blood-filtering organ that removes microbes and destroys old or damaged red blood cells.
It also makes disease-fighting components of the immune system (including antibodies and lymphocytes).
• Bone marrow is the spongy tissue found inside your bones. It produces the red blood cells our bodies need
to carry oxygen, the white blood cells we use to fight infection, and the platelets we need to help our blood
clot.
• The thymus filters and monitors your blood content. It produces the white blood cells called T lymphocytes

Q 76.C
• In yet another recognition of India’s commitment to protect and conserve the pristine coastal and
marine ecosystems through holistic management of the resources, the globally recognized and the
coveted International eco-label "Blue Flag”, has been accorded to two new beaches – Minicoy Thundi
Beach and Kadmat Beach- both in Lakshadweep. This takes the number of beaches certified under
the Blue Flag certification to twelve.
• Thundi Beach is one of the most pristine and picturesque beaches in the Lakshadweep archipelago where
white sand is lined by the turquoise-blue water of the lagoon. It is a paradise for swimmers and tourists
alike.
• Kadmat Beach is especially popular with cruise tourists who visit the island for water sports. It is a paradise
for nature lovers with its pearl white sand, blue lagoon waters, moderate climate, and friendly locals. Both
beaches have designated staff for beach cleanliness and maintenance; and for the safety and security of
swimmers. Both beaches comply with all the 33 criteria as mandated by the Foundation for Environment
Education (FEE).
• The other Indian beaches in the blue list are Shivrajpur-Gujarat, Ghoghla-Diu, Kasarkod and
Padubidri-Karnataka, Kappad-Kerala, Rushikonda (Andhra Pradesh), Golden (Odisha),
Radhanagar (Andaman and Nicobar), Kovalam (Tamil Nadu) and Eden (Puducherry) beaches.
Hence, option (c) is the correct answer.
Q 77.A
• Transfer pricing (TP) is basically an accounting practice that enables subsidiaries of the same
company to transact with each other. As an example, say Company Z has two subsidiaries, Firm A and
Firm B. Firm A and Firm B are distinct companies in their own right, and so cannot transfer resources or
services between each other for free. Transfer pricing is the method by which they ‘price’ these resources
when transacting with each other. Typically, the price of such transfers has to be at the market rate. Hence
statement 1 is correct.
• Transfer Pricing in India is governed under the Income Tax Act, of 1961. Hence statement 2 is not
correct.
• Similarly, arm’s length price (ALP) is a concept to do with the price subsidiaries charge each other. Section
92F(ii) of the Income Tax Act, 1961 defines arm’s length price as “a price which is applied or proposed to
be applied in a transaction between persons other than associated enterprises, in uncontrolled conditions”.
It basically means that one subsidiary should charge another the same price it would charge an unrelated
company. That is, it should hold its sibling company at arm’s length.
• For the most part, this is standard practice and does not raise any issues. However, there is a
possibility that transfer pricing can be used to evade tax. Using the same example, suppose Firm A
charges Firm B less than the market value for its services. In such a scenario, Firm A has a lower income
because it is getting less money for its services, but Firm B has a higher profit because it is paying less for
its inputs. The overall impact on the parent company, Company Z, is the same.
• Now, consider a situation where Firm A is in a high-tax country and Firm B is in a low-tax country. By
under-charging for its services, Firm A has ensured it pays less tax since it has lower revenues, while Firm
29 www.visionias.in ©Vision IAS
https://www.pdfnotes.co/

.
B also pays less tax on its higher profit because the tax rates are lower where it is located. Overall, the parent
company comes out of the transaction paying less taxes than it should.
• By establishing TP laws globally, the governments discourage evasion of taxes, as tax revenues are
important for the development of their economies. Indian Transfer Pricing Regulations (TP Regs)
extend to cross-border transactions between companies belonging to the same MNE group (i.e.,
controlled transactions). Some domestic transactions with companies obtaining tax deductions or eligible
for lower tax rates or incentives are also covered. All types of transactions - transfer of goods or services,
loans, guarantees, transfer, or remuneration for intangibles, sharing of expenses or income, etc. are covered
under TP Regs. The aim of TP regs is to provide a mechanism for determining whether the transfer price is
at arm’s length. The TP regs provide that the taxpayer may choose any of the six methods for determining
the ALP.

Q 78.C
• Recent Context: The celebration of Unying Aran, the new year festival of the Adi community, got
underway in the Siang district, Arunachal Pradesh. Hence option (c) is the correct answer.
• It is celebrated to mark the beginning of the Adi community’s new year i.e., the arrival of the spring season;
and to strengthen the bond among the community.
o Adi constitutes a major group and inhabits the lower part of the Lower Dibang Valley district especially
Roing and Dambuk areas.
o Adis basically depend on agriculture. Both wet rice cultivation and shifting cultivation are practiced.
o Padams, Milangs, Komkars, Minyongs, and Pasis collectively call themselves Adi meaning hill people.

Q 79.C
• Light Pollution
o The inappropriate or excessive use of artificial light – known as light pollution – can have serious
environmental consequences for humans, wildlife, and our climate.
• Components of light pollution include:
o Glare – excessive brightness that causes visual discomfort.
o Skyglow – brightening of the night sky over inhabited areas. Hence statement 1 is correct.
o Light trespass – light falling where it is not intended or needed.
o Clutter – bright, confusing, and excessive groupings of light sources.
• Affects of Light Pollution:
o Artificial light can wreak havoc on natural body rhythms in both humans and animals. Nocturnal
light interrupts sleep and confuses the circadian rhythm—the internal, twenty-four-hour clock that
guides day and night activities and affects physiological processes in nearly all living organisms.
o Studies show that light pollution is also impacting animal behaviors, such as migration patterns,
wake-sleep habits, and habitat formation. Because of light pollution, sea turtles and birds guided by
moonlight during migration get confused, lose their way, and often die. Large numbers of insects, a
primary food source for birds and other animals, are drawn to artificial lights and are instantly
killed upon contact with light sources (clutter). Birds are also affected by this, and many cities have
adopted a “Lights Out” program to turn off building lights during bird migration. Hence statement 2
is correct.

Q 80.B
• Eulophia obtuse or ground orchid - They have been receltly rediscovered in Dudhwa Tiger Reserve,
Uttar Pradesh after 118 years.
o It was last recorded in Pilibhit, Uttar Pradesh in 1902. The species is originally from Uttarakhand.
o Characteristics: It has white flowers and bright pink hues.
o Protection Status: Listed as “critically endangered” in the International Union for Conservation of
Nature (IUCN) Red List of endangered species.
• Researchers from the Regional Orchid Germplasm Conservation and Propagation Centre in Tinsukia and
the Botanical Survey of India in Itanagar have discovered the new orchid species – identified as
Schoenorchis mishmensis– named after the Mishmi Hills of Arunachal Pradesh, where it was found.
o In India, the genus Schoenorchis has seven species.
o The orchid is found on tree trunks at 900 metres in tropical mixed evergreen forests of Mishmi Hills in
Lower Dibang Valley district, the richest bio-geographical province of the Himalayan zone that falls
under one of the mega biodiversity hotspots of the world.
• Hence option (b) is the correct answer.

30 www.visionias.in ©Vision IAS


https://www.pdfnotes.co/

.
Q 81.C
• Mercury was known in Egypt and also probably in the East as early as 1500 BCE. The name mercury
originated in 6th-century alchemy, in which the symbol of the planet was used to represent the metal; the
chemical symbol Hg derives from the Latin hydrargyrum, “liquid silver.” Although its toxicity was
recognized at an early date, its main application was for medical purposes. Mercury is the only elemental
metal that is liquid at room temperature. Hence, statement 1 is correct.
• It alloys with copper, tin, and zinc to form amalgams or liquid alloys. An amalgam with silver is used as
a filling in dentistry. Mercury does not wet glass or cling to it, and this property, coupled with its rapid and
uniform volume expansion throughout its liquid range, making it useful in thermometers. Barometers and
manometers also used its high density and low vapour pressure. Hence, statement 2 is correct.
• The Bali Declaration on combating illegal trade in mercury was introduced by the Indonesian
Presidency during the Fourth meeting of the Conference of the Parties to the Minamata Convention on
Mercury (COP-4) held in Bali, Indonesia, 21 Mar 2022 - 25 Mar 2022 as a non-binding political declaration.

Q 82.D
• The first World Sloth Bear Day was observed on Wednesday to generate awareness and strengthen
conservation efforts around the unique bear species endemic to the Indian subcontinent. Classified
as vulnerable on the IUCN Red List, sloth bears are endemic to the Indian sub-continent and 90% of
the species population is found in India. Hence, statement 2 is not correct.
• The sloth bear is one of the eight bear species found across the globe. In the Indian subcontinent, they range
across the country through the Deccan plateau, coastal areas, western ghats, and to the base of the
Himalayas. This unique bear species is endemic to the Indian subcontinent. A tiny scattering of this
species occurs in Nepal and a sub-species in Sri Lanka thereby making India the main home for this
species. Hence, statement 1 is correct.
• The Wildlife (Protection) Act, of 1972, lists the sloth bear under Schedule I, granting this species the
same level of protection as tigers, rhinos, and elephants. However, this keystone species has mostly
been fighting a long uphill battle for survival and urgently deserves conservation and protection
measures. Hence, statement 3 is correct.

Q 83.C
• Closure Motion: It is a motion moved by a member to cut short the debate on a matter before the House. If
the motion is approved by the House, the debate is stopped forthwith and the matter is put to vote.
• There are four kinds of closure motions :
o Simple Closure: It is one when a member moves that the ‘matter having been sufficiently discussed be
now put to vote’.
o Closure by Compartments: In this case, the clauses of a bill or a lengthy resolution are grouped into
parts before the commencement of the debate. The debate covers the part as a whole and the entire part
is put to vote.
o Kangaroo Closure: Under this type, only important clauses are taken up for debate and voting
and the intervening clauses are skipped over and taken as passed. Hence option (c) is the correct
answer.
o Guillotine Closure: It is one when the undiscussed clauses of a bill or a resolution are also put to vote
along with the discussed ones due to want of time (as the time allotted for the discussion is over).

Q 84.A
• Before a loan account turns into an NPA (Non-Performing Asset), banks are required to identify incipient
stress in the account by creating three sub-categories under the Special Mention Account (SMA) category
as given in the table below:
o SMA sub-categories: SMA-0: Principal or interest payment not overdue for more than 30 days but
account showing signs of incipient stress.
o SMA-1: Principal or interest payment overdue between 31-60 Days.
o SMA-2: Principal or interest payment overdue between 61-90 Days.
• Categories of NPAs: Banks are required to classify non-performing assets further into the following three
categories based on the period for which the asset has remained non-performing and the realisability of the
dues:
o Substandard Assets
o Doubtful Assets
o Loss Assets

31 www.visionias.in ©Vision IAS


https://www.pdfnotes.co/

.
• Substandard Assets: With effect from March 31, 2005, a sub-standard asset is identified as one, which has
remained NPA for a period less than or equal to 12 months. Such an asset will have well-defined credit
weaknesses that jeopardize the liquidation of the debt and are characterized by the distinct possibility that
the banks will sustain some loss if deficiencies are not corrected.
• Doubtful Assets: With effect from March 31, 2005, an asset is classified as doubtful if it has remained in
the sub-standard category for a period of 12 months. A loan classified as doubtful has all the weaknesses
inherent in assets that were classified as sub-standard, with the added characteristic that the weaknesses
make collection or liquidation in full, - on the basis of currently known facts, conditions and values – highly
questionable and improbable.
• Loss Assets: A loss asset is one where loss has been identified by the bank or internal or external auditors
or the RBI inspection but the amount has not been written off wholly. In other words, such an asset is
considered uncollectible and of such little value that its continuance as a bankable asset is not warranted
although there may be some salvage or recovery value.
• Thus, Special Mention Accounts carry the least risk as they are not non-performing assets. Among
the other 3 categories which are classified as NPAs, loss assets carry the highest risk, while substandard
assets carry the lowest. Hence option (a) is the correct answer.

Q 85.A
• The pravrajya ceremony marked a person’s going forth from home into homelessness and his/her
becoming a novice under a preceptor. Although this term occurs in the early Brahmanic tradition of the
Upaniṣads, it is also applicable to Buddhist and Jain monks, as well as to Hindu saṃnyāsins. The Pāli
equivalent is paribbājaka. Hence statement 1 is correct.
• Monks renounced social obligations to take on an alternative life when they joined the Order. They lived as
equal members of the Order, denying caste distinctions. But they lived in monasteries near villages and
towns so that they could draw on the support of the lay community, namely, those who were Buddhists or
Jainas but were not initiated into renunciatory groups. Lay followers were referred to as upasaka (male
followers) and upasika (female followers).
o An upasaka/upasika was a lay follower of Buddhism but who had not taken monastic vows. hence
statement 2 is not correct.
• The monastic order of monks, and ultimately also of nuns, was created within the Buddha’s lifetime.
o With its establishment, followers of the Buddha were able to carve out a distinct identity for themselves
within the larger community of renunciants.
o The Buddhist sangha became a core institution and a major factor in the dissemination of the Buddha’s
doctrine.
o The Vinaya Pitaka gives an account of the establishment of the sangha and the rules that governed
it.

Q 86.C
• Article 12 of the Indian constitution defines the State which includes the Government and Parliament of
India and the Government and the Legislature of each of the States and all local or other authorities within
the territory of India or under the control of the Government of India.
• So, Article 12 defines the state and is only applicable to parts 3 and 4 of the constitution. Article 12 defines
4 authorities to be a part of the state, which are:
o Government and Parliament of India,
o Government and legislature of State,
o Local Authority, Hence option 4 is correct.
o And other authorities within the territory of India or under the control of the Government of India.
• BCCI in India is a registered society. Pradeep Kumar Biswas v. Indian Institute of Chemical Biology, (2002)
5 SCC 111, there was a 7-judge bench which defined a state as a Body having deep and pervasive State
control over its functions, finances and administration; Mere regulatory control of the government over the
body is not sufficient.
• In a very famous case of Zee Telefilms Ltd v. Union of India, AIR 2005 SC 2677, there was a question
as to whether BCCI will come under the category of State under Article 12. The judgement stated
that BCCI is not 'State' on the finding that the requisites of Biswas judgement are not fulfilled; Yet,
when a private body exercises public functions (public duty) even if not State, the aggrieved person
has a remedy under ordinary law and also under Article 226. Hence option 1 is not correct.
• In another case, Janet Jayepaul v. SRM University, AIR 2016 SC 73; the University discharges
‘important public functions’ hence, State; therefore, amenable to writ jurisdiction and hence
University was considered under the definition of State. Hence option 2 is correct.
32 www.visionias.in ©Vision IAS
https://www.pdfnotes.co/

.
• Whether the judiciary falls under the meaning of the State under Art 12?
o This question arises because Art 12 makes an apparent exclusion of the judiciary from the State. It
makes specific mention of the executive and legislative organs and leaves out the judiciary, which
generally falls under the ordinary meaning of State.
o Through the cases of Naresh Sridhar Mirajkar v. State of Maharashtra, A R Antulay v. R S
Nayak and Riju Prasad Sharma v. State of Assam, it has been established that the judiciary does
not fall under the meaning of the State under Art 12 while working on its judicial side. Hence
option 3 is not correct.
o Therefore, wrong determinations of courts are not amenable to writ jurisdiction. They can, however, be
challenged through a petition for appeal, review or revision. The judiciary, however, does fall under the
meaning of the State under Art 12 while operating on its administrative and rule-making side. Hence,
any violation of Part III by any administrative decision or any rules made by the judiciary may be
challenged by a petition under Arts 32 or 226.

Q 87.B
• The term "capital intensive" refers to business processes or industries that require large amounts of
investment to produce a good or service and thus have a high percentage of fixed assets, such as property,
plant, and equipment (PP&E). Companies in capital-intensive industries are often marked by high levels of
depreciation.
o Labor intensive refers to a process or industry that requires a large amount (investment) of labor to
produce its goods or services. Hence statement 1 is not correct.
• A capital-intensive production process will have a relatively low ratio of labor inputs and will have higher
labor productivity (output per worker). Hence statement 2 is correct.
• Capital-intensive industries tend to have high levels of operating leverage, which is the ratio of fixed costs
to variable costs. As a result, capital-intensive industries need a high volume of production to provide an
adequate return on investment.
• In recent years, technological development has enabled increased capital intensity in many industries. New
technology such as Artificial Intelligence, micro-computers, and the internet has enabled previously labor-
intensive industries (taxis, delivery to become more capital-intensive – e.g. with the rise of self-driving cars.
• Increased capital intensity can cause some workers to lose their jobs because they are no longer needed.
This can lead to structural unemployment – at least in the short term. However, more capital-intensive
industries create different kinds of jobs. There are new jobs in the design of software, AI, and marketing.
Also, capital-intensive production can lead to lower prices and higher incomes. This causes an increased
demand for a greater variety of services in the economy.

Q 88.D
• The Protection of Women from Sexual Harassment at Workplace Act, 2013, defined sexual harassment,
laying down the procedures for a complaint and inquiry, and the action to be taken. It defines sexual
harassment, lays down the procedures for a complaint and inquiry, and the action to be taken. It was the
result of Vishakha guidelines laid down by the Supreme Court in 1997. Hence statement 1 is correct.
• Highlights of the Act:
o The Act defines sexual harassment at the workplace and creates a mechanism for redressal of
complaints. It also provides safeguards against false or malicious charges.
o Every employer is required to constitute an Internal Complaints Committee at each office or
branch with 10 or more employees. Hence statement 2 is correct.
o The District Officer is required to constitute a Local Complaints Committee at each district, and if
required at the block level.
o The ICC has powers similar to those of a civil court in respect of summoning and examining any
person on oath, and requiring the discovery and production of documents. Hence statement 3 is
correct.
o The Complaints Committees are required to provide for conciliation before initiating an inquiry if
requested by the complainant.
o The ICC may either forward the victim’s complaint to the police, or it can start an inquiry that has to
be completed within 90 days.

Q 89.C
• The Non-cooperation Movement was launched on 5th September 1920 by the Indian National
Congress (INC) under the leadership of Mahatma Gandhi.
• In September 1920, in a Congress session in Calcutta, the party introduced the Non-Cooperation program.
33 www.visionias.in ©Vision IAS
https://www.pdfnotes.co/

.
• It signified a new chapter in the history of the Indian freedom struggle.
• The Non-Cooperation Movement was launched in the wake of a series of events including the Jallianwala
Bagh Massacre and was called off because of the Chauri-Chaura incident of 1922.
• The Non-cooperation movement was launched by Mahatma Gandhi in 1920 with the aim of self-
governance.
• Indian National Congress outlined a seven-item program of non-cooperation-
o Surrender of titles and honorary offices,
o Refusal to attend Government durbars and officials' function,
o Boycott of government or government-aided schools and colleges,
o Boycott of British courts,
o Refusal to all classes to offer themself for service in Mesopotamia,
o Boycott of elections to provincial and Central assembly elections, and
o Boycott of foreign goods.
• It was also planned that if the above steps did not bring results, people would refuse to pay their taxes.
• The INC also demanded Swarajya or self-government.
• Only completely non-violent means would be employed to get the demands fulfilled.
• The non-cooperation movement was a decisive step in the independence movement because, for the first
time, the INC was ready to forego constitutional means to achieve self-rule.
• Gandhiji had assured that Swaraj would be achieved in a year if this movement was continued to
completion.
• Gandhiji called off the movement in February 1922 in the wake of the Chauri Chaura incident.
• In Chauri Chaura, Uttar Pradesh, a violent mob set fire to a police station killing 22 policemen during a
clash between the police and protesters of the movement.
• Gandhiji called off the movement saying people were not ready for revolt against the government through
ahimsa. Several leaders like Motilal Nehru and C R Das were against the suspension of the movement
only due to sporadic incidents of violence.
• Hence, option (c) is the correct answer.

Q 90.D
• Puisne is a French word meaning "later born" or "younger" and refers to the seniority of the rank of
judges. In common law countries, a puisne judge is any judge ranked lower in seniority, who is not the
court's Chief Justice. Common law is the kind of law that judges create through written opinions rather than
through laws or constitutions (statutory law). Hence, statement 1 is correct.
• In the United Kingdom, "puisne judges" refer to judges who are not Lord Chancellor, Lord Chief
Justice of England, or Master of the Rolls, as defined by the Supreme Court of Judicature Act of 1877. They
are considered judges of the High Court. In India, the term is used for reference during seniority-
based appointments and elevations to High Courts, but it does not impact a judge's exercise of judicial
power as all judges have equal powers. The court's chief justice has an additional administrative role as the
senior-most judge. Hence, statement 2 is correct.

Q 91.A
• The Directorate of Enforcement is a multi-disciplinary organization mandated to investigate offenses
of money laundering and violations of foreign exchange laws. The CBI is the premier investigating
agency of India.
• It is composed of officers from the Indian Revenue Service, Indian Police Service, and the Indian
Administrative Service as well as promoted officers from its cadre. In addition to directly hiring people, the
Directorate also draws officers from different Investigating Agencies like Customs & Central Excise,
Income Tax, Police, etc. on deputation.
• Unlike CBI, ED cannot register a case on its own. It is required by the agencies such as CBI or state
police to register an offense based on which the Case Information Report is filed by Enforcement
Directorate. Recently, the Supreme Court upheld the power of the Enforcement Directorate to arrest under
PMLA. The SC bench held that the power given to ED for making arrests, conducting search and
seizures, and attaching proceeds of crime are constitutionally valid and do not suffer from the vice of
arbitrariness. Hence statement 1 is correct.
• The Fugitive Economic Offenders Act, 2018 (FEOA)
• This law was enacted to deter economic offenders from evading the process of Indian law by remaining
outside the jurisdiction of Indian courts. It is a law whereby the Directorate is mandated to attach the
properties of the fugitive economic offenders who have escaped from India warranting arrest and provide
for the confiscation of their properties to the Central Government. While the ED works under the
34 www.visionias.in ©Vision IAS
https://www.pdfnotes.co/

.
administrative control of Department of Revenue, Ministry of Finance, CBI functions under the overall
supervision of the Ministry of Personnel, Public Grievances and Pensions. Hence statement 2 is not
correct.

Q 92.B
• The term “open ballot system” is used to describe votes in which participants’ choices are not confidential.
It is the opposite of a secret ballot. In most legislatures around the world, elected representatives vote on
proposed legislation using an open ballot system, which is often done by members of parliament simply
assembling on opposing sides of the chamber. One side represents the “no” vote and the other the “yes”
vote. This system is widespread in politics as it enables voters to hold their representatives to account for
what they voted for and against.
• Open ballot voting applies in the election to the Council of States only. Hence option 1 is correct.
o Every political party which has its member(s) as MLAs can appoint an authorized agent to verify as to
whom its members have voted for.
o The authorized agent will be seated inside the polling station in seats provided by the Returning Officer
(RO).
o In the case of MLAs who are members of political parties, after they mark the vote and before inserting
the ballot box, are required to show the marked ballot paper to the authorized agent of their party.
• As per Article 55(3) of the Constitution of India, the election of the President shall be held in accordance
with the system of proportional representation by means of a single transferable vote and the voting at
such election shall be by secret ballot. Hence option 3 is not correct.
• The Vice-President is elected by an electoral college consisting of members of both Houses of Parliament,
in accordance with the system of proportional representation by means of the single transferable vote and
the voting in such election is by secret ballot. Hence option 4 is not correct.
• With the complaints from several states that cross voting by MLAs led to corrupt practices during election
of Legislative Council members due to secret ballot system, the government has decided to bring an
amendment to Representation of Peoples Act to make the election transparent. Hence option 2 is not
correct.

Q 93.C
• ‘Healthy and Energy Efficient Buildings’ Initiative for Workplaces: With an aim to make workplaces
healthier and greener, Energy Efficiency Services Ltd (EESL) jointly with the US Agency for
International Development’s (USAID) MAITREE program, has launched the “Healthy and Energy
Efficient Buildings” initiative. This initiative was launched on the occasion of World Environment Day.
Hence, statement 1 is correct.
• The Market Integration and Transformation Program for Energy Efficiency (MAITREE), under which this
initiative has been launched is aimed at accelerating the adoption of cost-effective energy efficiency as a
standard practice within buildings, and specifically focuses on cooling and EESL has taken the leadership
by being the first to implement this framework in its own offices.
• “Healthy and Energy Efficient Buildings” Initiative will address the challenges of retrofitting existing
buildings and air conditioning systems so that they are both healthy and energy-efficient. Hence,
statement 2 is correct.

Q 94.B
• Theyyam, also known as Kaliyattam, is a ritual dance popular in north Kerala. Hence pair 1 is
correctly matched.
o It encompasses dance, mime and music. It exalts the beliefs of the ancient tribals, who gave a lot of
importance to the worship of heroes and the spirits of their ancestors.
o The ceremonious dance is accompanied by the chorus of such musical instruments as Chenda,
Elathalam, Kurumkuzal and Veekkuchenda.
o There are over 400 separate Theyyams, each with their own music, style and choreography.
o Each artist represents a hero with great power. Performers wear heavy make-up and adorn flamboyant
costumes.
• Kambala is an annual buffalo race event popular in coastal Karnataka districts during the winter
when contestants sprint along waterlogged muddy fields holding a rope attached to two buffaloes.
Hence pair 2 is not correctly matched.
o It is believed that the race is held to propitiate the Gods for a good harvest, besides being a recreational
sport for farmers.
o The length of the race is usually about 100 meters.
35 www.visionias.in ©Vision IAS
https://www.pdfnotes.co/

.
o The contest generally takes place between two pairs of buffaloes, controlled by a feverishly whip-
lashing farmer.
• Yakshagana is a traditional theatre, developed in Dakshina Kannada, Udupi, Uttara Kannada,
Shimoga and western parts of Chikmagalur districts, in the state of Karnataka and in Kasaragod district in
Kerala. Hence pair 3 is correctly matched.
o The performance combines dance, music, dialogue, costume, make-up, and stage techniques. It is
believed to have evolved from pre-classical music and theatre. It is sometimes simply called ‘Aata’ or
‘āṭa’ (meaning ‘the play’). This theatre style is mainly found in coastal regions of Karnataka correct.
Yakshagana is traditionally presented from dusk to dawn. Its stories are drawn from Ramayana,
Mahabharata, Bhagavata and other epics from both Hindu, Jain and other ancient Indic traditions.

Q 95.D
• Gross Domestic Climate Risk Report
o Recent Context: A recent report has warned that the Earth's climate may be on the edge of permanent
change due to 'dangerous' feedback loops.
o 9 out of 50 regions in the world facing high climate risk to a fragile physical infrastructure, fall in India.
o The report was released by the Cross Dependency Initiative (XDI). Hence, option (d) is the correct
answer.
o XDI is a global organization specializing in climate risk analysis for regions, banks, and companies.
o The index calculated the physical climate risk to built environments such as buildings and properties.
o Physical risk refers to vulnerability from 8 climate change events like heat waves, coastal flooding,
extreme wind, etc.
o ADR - The index also assigned an Aggregated Damage Ratio (ADR) to each region.
o ADR signifies the total amount of damage a region’s-built environment would sustain in 2050 and a
high ADR signifies more peril.
o Assam, Bihar, and Tamil Nadu had the highest ADR among the Indian States.

Q 96.B
• Marrakech Partnership for Global Climate Action is a partnership between States and non-state actors,
launched during COP22 to the United Nations Framework Convention on Climate Change was held from
7 to 18 November 2016 in Marrakesh, Morocco. Hence, statement 1 is not correct.
• The Marrakech Partnership for Global Climate Action supports the implementation of the Paris Agreement
by enabling collaboration between governments and the cities, regions, businesses and investors that
must act on climate change. Its main objective is to strengthen collaboration between governments and key
stakeholders to immediately lower emissions and increase resilience against climate impacts. Hence,
statement 2 is correct.
• The Marrakech Partnership has three principal reporting and tracking vehicles for capturing the breadth
of climate action globally: the Global Climate Action portal, the Yearbook of Global Climate
Action and the Summary for Policymakers. In addition, a periodical report is published on the achievements
of the High-Level Champions and the Marrakech Partnership.

Q 97.C
• By the year 1526, the Bahmani kingdom had disintegrated into five independent sultanates. They were
Ahmadnagar, Bijapur, Berar, Golkonda and Bidar and were known as Deccan Sultanates by the region
within which they emerged.
• From 1490 to 1633, the Nizam Shahi dynasty ruled the kingdom of Ahmadnagar in India's Deccan.The
founder was Malik Aḥmad, who in 1490 fixed his capital on a new site called Ahmadnagar after himself.
o The kingdom lay in the northwestern Deccan, between the states of Gujarat and Bijapur. It secured the
great fortress of Daulatabad in 1499 and added Berar in 1574. Hence pair 1 is correctly matched.
• The Adil Shahi dynasty (1489–1686) was the ruling family of the kingdom of Bijapur, one of the two
principal successor states to the Muslim sultanate of Bahmani in the Deccan. It was named after its founder,
Yusuf Adil Shah, one of the provincial governors under the Bahmani kingdom and a protege of Mahmud
Gavan.
o The dynasty strongly resisted the Mughal advance southward in the 17th century until it was
extinguished by the Indian emperor Aurangzeb with the capture of Bijapur in 1686. Hence pair 2 is
correctly matched.
• The Qutb Shahi dynasty (1518–1687), rulers of the kingdom of Golconda in the southeastern Deccan of
India, was one of the five successor states of the Bahmani kingdom. The founder was Quli Qutb Shah, a

36 www.visionias.in ©Vision IAS


https://www.pdfnotes.co/

.
Turkish governor of the Bahmani eastern region, which largely coincided with the preceding Hindu state of
Warangal.
o Qutb Shah declared his independence in 1518 and moved his capital to Golconda. Toward the end of
the century, Muhammad Quli Qutb Shah built a new capital at Hyderabad, a few miles away. Hence
pair 3 is correctly matched.
• The Imad Shahi Sultanate of Berar (1490-1572), in Maharashtra, was one of the five Deccan Sultanates.
The Imad Shahi dynasty was established by Fathullah Imad Shah (1490–1504), with its capital at Ellichpur.
• The Farooqi dynasty was the ruling dynasty of the Khandesh Sultanate from its inception in 1382 until
its annexation by the Mughal emperor Akbar in 1601. The founder of the dynasty, Malik Ahmad (also
known as Malik Raja), participated in a rebellion against the Bahmani ruler Muhmmad Shah I in his early
years. Hence pair 4 is not correctly matched.

Q 98.C
• During the Sultanate period, the village headman was known as the Muqqaddam whereas the smaller
landowners were known as khuts.
• In addition to their own holdings, they held lands for which they paid revenue at concessional rates. They
also worked in conjunction with the governor in the collection of revenue and maintaining law and order,
etc. However, under Alauddin Khilji's regime, they were made to pay the same taxes as others.
• Hence option (c) is the correct answer.

Q 99.C
• Right of Persons with Disabilities Act 2016:
• Disability has been defined based on an evolving and dynamic concept.
o The types of disabilities have been increased from 7 to 21.
o The act added mental illness, autism, spectrum disorder, cerebral palsy, muscular dystrophy, chronic
neurological conditions, speech and language disability, thalassemia, hemophilia, sickle cell disease,
multiple disabilities including deaf blindness, acid attack victims and Parkinson’s disease which
were largely ignored in earlier act. Hence statement 1 is correct.
• The Union Government has been authorized to notify any other category of specified disability. Hence
statement 2 is not correct.
• All children with benchmark disabilities between 6 and 18 years shall have the right to free education.
• The act increased the reservation for people with benchmark disabilities from 3% to 4% in
government jobs and from 3% to 5% in higher education institutes. Hence statement 3 is correct.

Q 100.C
• Recent Context: Financial Action Task Force (FATF) suspended Russia's membership over Ukraine
war.
o Although suspended, Russia remains a FATF member. However, it will be barred from attending
meetings and accessing documents. Also, FATF updated its list for Jurisdictions Under Increased
Monitoring (i.e. Grey list) and High-Risk Jurisdictions (i.e. Black List). Hence option (c) is the
correct answer.
• The Financial Action Task Force (on Money Laundering) (FATF), also known by its French name,
Groupe d'action financière (GAFI), is an intergovernmental organisation founded in 1989 on the
initiative of the G7 to develop policies to combat money laundering.
o In 2001, its mandate was expanded to include terrorism financing.
o The objectives of FATF are to set standards and promote effective implementation of legal, regulatory
and operational measures for combating money laundering, terrorist financing and other related threats
to the integrity of the international financial system. FATF is a "policy-making body" that works to
generate the necessary political will to bring about national legislative and regulatory reforms in these
areas.
o FATF monitors progress in implementing its Recommendations through "peer reviews" ("mutual
evaluations") of member countries.
o FATF Blacklist: Officially known as High-Risk Jurisdictions subject to a Call for Action, the FATF
blacklist sets out the countries that are considered deficient in their anti-money laundering and
counter-financing of terrorism regulatory regimes.
✓ The list is intended to serve not only as a way of negatively highlighting these countries on the
world stage, but as a warning of the high money laundering and terror financing risk that they
present.

37 www.visionias.in ©Vision IAS


https://www.pdfnotes.co/

.
✓ It is extremely likely that blacklisted countries will be subject to economic sanctions and other
prohibitive measures by FATF member states and other international organizations.
o FATF Greylist: In addition to its blacklist, the FATF also issues a grey list, officially referred to
as Jurisdictions Under Increased Monitoring.
✓ Like the blacklist, countries on the FATF grey list represent a much higher risk of money
laundering and terrorism financing but have formally committed to working with the FATF
to develop action plans that will address their AML/CFT deficiencies.
✓ Countries on the list may face economic sanctions from institutions like the IMF and the World
Bank and experience adverse effects on trade.

Copyright © by Vision IAS


All rights are reserved. No part of this document may be reproduced, stored in a retrieval system or transmitted
in any form or by any means, electronic, mechanical, photocopying, recording or otherwise, without prior
permission of Vision IAS.

38 www.visionias.in ©Vision IAS


https://www.pdfnotes.co/

www.upscmaterial.online

Join Our Official Telegram Channel

https://telegram.me/+YwhD2COlqds3N2I1

Or can Code or Join Our Channel

You might also like